evidence exam

¡Supera tus tareas y exámenes ahora con Quizwiz!

A plaintiff sued a defendant for defamation, asserting in her complaint that the defendant had called the plaintiff a thief in front of a number of business associates. The plaintiff calls two witnesses to the stand, both of whom testify that they heard the defendant refer to the plaintiff as a thief in front of the business associates. The plaintiff does not take the stand herself. The defendant pleads truth of the statement as an affirmative defense and calls a witness to the stand. The defense witness is prepared to testify that he was a co-worker of the plaintiff when the plaintiff supplemented her income by tending bar three nights a week. The witness will testify that he saw the plaintiff take a $20 bill from the tavern's cash register and secrete the money in her pocket. The plaintiff's attorney objects. May the defense witness's testimony be allowed? a. yes, as substantive evidence that the plaintiff is, in fact, a thief b. yes, because theft is a crime indicating dishonesty c. no, because specific bad acts may not be used to show bad character d. no, because the plaintiff never took the stand

a. is correct. The defense witness's testimony is admissible character evidence because the plaintiff's character is directly in issue in the case. As a general rule, evidence of character to prove the conduct of a person in the litigated event is not admissible in a civil case. However, when proof of a person's character, as a matter of substantive law, is an essential element of a claim or defense in a civil action, character evidence is admissible because it is the best method of proving the issue. Under the Federal Rules, any of the types of evidence-reputation, opinion, or specific acts-may be used. Here, character is an issue in the plaintiff's defamation action because the defendant has pleaded as an affirmative defense that his statement claiming that the plaintiff is a thief is the truth. The defense witness's testimony that he saw the plaintiff take the money from the cash register is relevant because it tends to show that the defendant spoke the truth. Hence, it should be allowed. (B) is incorrect because the fact that the theft here could be considered a crime of dishonesty would be relevant only if the plaintiff's credibility were being impeached, and only then if proof of an actual conviction were provided. Here, the testimony is admissible because it is being offered as substantive evidence of an aspect of the plaintiff's character that is an essential element of a defense in the case. (C) is incorrect. One of the few cases where testimony as to specific acts of a person may be used to show that person's character is when character itself is one of the essential issues in the case, as it is here. (D) is incorrect because the fact that the plaintiff never took the stand only means that she has not placed her credibility in issue and become subject to impeachment. Here, however, the plaintiff's character is in issue and the testimony is being offered as substantive evidence of her character rather than to impeach her credibility.

A plaintiff brought an action against a defendant for property damages, alleging that the defendant's car nicked the side of the plaintiff's truck while the defendant was changing lanes on an expressway. At trial, the defendant sought to introduce evidence of her good driving record. Is the evidence admissible? a. no, because it is improper character evidence b. no, because it is self-serving c. yes, because it is proper character evidence d. yes, because it is habit evidence

a. is correct. The driving record is inadmissible because it is being offered as character evidence. In a civil case, evidence of character to prove the conduct of a person in the litigated event is generally not admissible. The slight probative value of character is outweighed by the dangers of prejudice and distracting the jury from the main issues. Therefore, circumstantial use of prior behavior patterns for the purpose of drawing the inference that a person has a particular character trait and that, at the time and place in question, she probably acted in conformity with it is not permitted. Evidence of the defendant's good driving record is being offered to show that she is a careful driver and to raise the inference that, when the accident occurred, she was acting in conformity with that trait. This constitutes impermissible use of character evidence and is inadmissible. (B) is incorrect because evidence is not excludable because it is self-serving. Virtually all evidence is self-serving to the party offering it. (C) is incorrect because it is based on the mistaken assumption that character evidence is admissible. As stated above, character evidence is generally inadmissible in a civil case. It is admissible in a civil case only when proof of a person's character, as a matter of substantive law, is an essential element of a claim or defense (e.g., in a defamation case). The defendant's character is not in issue, so the driving record is inadmissible. (D) is incorrect because this is not habit evidence. Habit describes one's regular response to a specific set of circumstances. Character describes one's disposition with respect to general traits. The defendant's good driving record describes a general behavior pattern of careful driving, rather than a regular response to a specific set of circumstances. Thus, this is character evidence, rather than habit evidence.

Miles is on trial for running a dog fighting ring. The prosecutor alleges that Miles owned a compound outside the city where he trained dozens of dogs to attack other dogs. Miles would then allegedly host large dog-fighting parties at the compound, where he would force his dogs to fight each other in a large pit and charge his guests to watch. In his defense, Miles calls his wife Janet to the stand. Janet testifies that she has known Miles for five years, and that at home he never showed any cruelty to animals, and was always kind to his own pets. Assume that the prosecutor has a witness named Sarah who was at Miles' and Janet's house a year ago and saw Miles kick his pet dog in the head after the dog did not obey his command. Sarah also reports that Janet was present and saw Miles kick the dog. How (if at all) can the prosecutor use this information at trial? a. Ask Janet on cross-examination: "Isn't it true that you once saw Miles kick your pet dog in the head when the dog did not obey his commands?" b. Call Sarah to the stand in the prosecutor's case-in-chief to testify she saw Miles kick his pet dog in the head when the dog did not obey his command. c. Ask Janet a question on cross-examination about whether she ever saw Miles kick a dog, and call Sarah to the stand in rebuttal to testify about the event. d. The prosecutor is not allowed to ask Janet this question on cross-examination or call Sarah to the stand to testify about the event.

a. is correct. Although specific instances of conduct are not allowed to prove character on direct examination, they are permitted on cross-examination to challenge the opinion testimony of a character witness. b. is incorrect. Although the prosecutor is permitted to call his own character witness to respond to Miles' character witness, the character witness can only testify using opinion or reputation evidence. Specific instances of conduct are not permitted on direct examination to prove character. c. is incorrect. Although the prosecutor is permitted to call his own character witness to respond to Miles' character witness, the character witness can only testify using opinion or reputation evidence. Specific instances of conduct are not permitted on direct examination to prove character. d. is incorrect. Although specific instances of conduct are not allowed to prove character on direct examination, they are permitted on cross-examination to challenge the opinion testimony of a character witness.

A police officer pulled over Sam's car after seeing it swerve numerous times into the oncoming lane of traffic. The officer ordered Sam out of the car and told him to take a Breathalyzer test. Sam refused. Based on Sam's erratic driving and Sam's refusal to take the test, the officer arrested Sam for driving while under the influence of alcohol. A person is guilty of this crime if his ability to drive a car is impaired because of consumption of alcohol. At trial, the prosecutor wants the officer to testify that Sam refused to take the Breathalyzer test. Is this evidence relevant? a. Yes, because his refusal to take the test makes it more probable that he was in fact intoxicated. b. Yes, because his refusal to take the test is sufficient to prove that he was in fact intoxicated. c. No, because he may have had many legitimate reasons for not wanting to take the test. d. No, because there is no other evidence that he was intoxicated except for his swerving, and his swerving may be unrelated to intoxication.

a. is correct. Although there are other reasons why he may have refused to take the test, the legal standard for relevance is very low—the proffered evidence only needs to have some tendency to make a fact in consequence more or less probable. Since refusing to take the Breathalyzer may be probative of consciousness of guilt, the evidence is relevant. b. is incorrect. This is not the correct standard for relevance—the party offering the evidence need not argue that the evidence is sufficient to prove a fact in consequence, only that the evidence makes it more likely that a fact in consequence is true c. is incorrect. Just because there are other innocent reasons for his conduct does not mean that the conduct does not have some probative value in determining whether he is guilty. As long as his refusal to take the test has some tendency to make a fact in consequence more likely, it is relevant. d. is incorrect. There is no requirement that evidence be corroborated in order to be relevant. As long as the evidence makes a fact in consequence more likely, it is relevant.

During a dispute in a bar, Charlie pulled out a knife and stabbed Harry in the chest. Harry died instantly. Charlie admits that he stabbed Harry in the chest, but he pleads self-defense, arguing that Harry swung at his face with a broken beer bottle first, and that he reasonably believed that he needed to use deadly force to protect himself. The prosecutor has a character witness who will say that Charlie has a reputation in the bar for being a very violent man. In his defense, Charlie wishes to testify that a few weeks before the incident, a friend of his had told him that Harry had stabbed two other people in the neck with a broken beer bottle in bar fights. How should the judge rule on the prosecutor's character witness and on Charlie's testimony? a. Allow Charlie to testify about Harry's previous stabbing of two other people, but give a limiting instruction explaining that the jury should only use the testimony as evidence about Charlie's state of mind at the time of the stabbing, not as evidence of Harry's propensity for violence. The court should preclude the prosecutor from admitting evidence about Charlie's reputation for violence, either in the prosecutor's case-in-chief or in its rebuttal. b. Allow Charlie to testify about Harry's previous stabbing of two other people, but give a limiting instruction explaining that the jury should only use the testimony as evidence about Charlie's state of mind at the time of the stabbing, not as evidence of Harry's propensity for violence. After Charlie testifies as to the previous stabbings, the court should then allow the prosecutor in rebuttal to admit evidence about Charlie's reputation for violence. c. Allow Charlie to testify about Harry's previous stabbing of two other people, and allow the jury to consider the evidence for any relevant purpose. The court should preclude the prosecutor from admitting evidence about Charlie's reputation for violence in its case-in-chief and in rebuttal. d. Allow Charlie to testify about Harry's previous stabbing of two other people, and allow the jury to consider the evidence for any relevant purpose. After Charlie testifies as to the previous stabbings, the court should then allow the prosecutor in rebuttal to admit evidence about Charlie's reputation for violence.

a. is correct. Charlie's evidence about Harry's previous stabbings would be inadmissible to prove Harry's propensity for violence, because under Rule 405, propensity can only be proven by reputation or opinion evidence, not specific act evidence. However, because Charlie knew about these stabbings, it is relevant to prove that he acted with a reasonable belief that deadly force was necessary, and so it admissible for that non-propensity purpose. Because Charlie is not offering this as propensity evidence, he is not "opening the door" to prosecution propensity evidence, and the prosecutor cannot admit evidence of Charlie's propensity for violence. b. is incorrect. Charlie is not offering the evidence of his knowledge of Harry's previous stabbings as propensity evidence, so he is not "opening the door" to prosecution propensity evidence, and the prosecutor cannot admit evidence of Charlie's propensity for violence. c. is incorrect. Charlie's evidence about Harry's previous stabbings is inadmissible to prove Harry's propensity for violence, because under Rule 405, propensity can only be proven by reputation or opinion evidence, not specific act evidence. However, because Charlie knew about these stabbings, it is relevant to prove that he acted with a reasonable belief that deadly force was necessary. Therefore it admissible for that non-propensity purpose, and a limiting instruction must be given to the jury to ensure they do not misuse the testimony. d. is incorrect. Charlie's evidence about Harry's previous stabbings is inadmissible to prove Harry's propensity for violence, because under Rule 405, propensity can only be proven by reputation or opinion evidence, not specific act evidence. Thus, Charlie is not allowed to offer the evidence of his knowledge of Harry's previous stabbings as propensity evidence. Since this evidence is inadmissible to prove Harry's propensity for violence, Charlie is not offering any propensity evidence, and therefore he is not "opening the door" to prosecution propensity evidence, and the prosecutor cannot admit evidence of Charlie's propensity for violence.

Fred is on trial for selling heroin. He has a prior conviction for burglary from two years ago. Under what conditions is it most likely that the prior conviction for burglary will be admitted in evidence? a. If Fred testifies. b. If Fred calls a character witness to testify that he has known Fred for ten years and in his opinion Fred would never use or sell heroin. c. If the prosecutor calls a character witness to testify that she has known Fred for ten years and in her opinion Fred has a propensity to commit crimes. d. If the burglary conviction came from a jury verdict rather than a plea bargain.

a. is correct. If Fred testifies, he puts his credibility at issue in the case. He thus opens the door to the possibility that the prosecutor will impeach him with a prior conviction under Rule 609. The prior conviction still may not be admissible—the judge will have to determine that the probative value of the evidence for impeachment outweighs the unfair prejudice to the defendant—but at least there is a possibility that the burglary conviction will be admitted. b. is incorrect. Under Rule 608(b), if the character witness testifies, he can be cross-examined with any specific acts that Fred may have committed which contradict the opinion the expert has about Fred. But here the character witness did not testify that Fred never committed any crimes, or that Fred never committed burglary. The character witness simply testified that Fred never used or sold heroin. Fred's prior conviction for burglary does not contradict that opinion and so is irrelevant for impeaching the character witness. c. is incorrect. The prosecution's witness (who could only testify if Fred called his own character witness first) could only testify as to her opinion of Fred's character or Fred's reputation. Rule 608(b) states that she could not testify as to specific acts that he has done. The defense attorney could ask about specific acts on cross-examination, but only specific acts that contradict the character witness' testimony, not acts which support the character witness' testimony. d. is incorrect. The admissibility of a prior criminal conviction is treated the same way under the rules of evidence whether it is the result of jury verdict or a plea bargain.

Sarah's boyfriend George was killed in his own kitchen with a knife early one morning. He had a knife in his hand. When the police responded to the call, they found Sarah in the kitchen crying, and the back door to the house was open. When the police asked Sarah what happened, she said nothing. The police continued their investigation over the next two weeks, and came to the conclusion that Sarah's grown son Andrew had killed George. The police arrested Andrew and charged him with murder. At trial, Andrew called his mother Sarah as a witness. She testified that she was the one who killed George, and that she acted in self-defense. She testified that George had come home drunk and began arguing with her and then grabbed a knife and swung it at her chest, and that she was able to grab her own knife and stab him in the chest. She also testified that she washed the knife she used and put it back in the drawer before the police arrived. She further testified that her son Andrew was never present that night. On cross-examination, the prosecutor asked Sarah if it was true that she said nothing when the police originally asked her what happened. Is this question permissible? a. Yes, the question is admissible to impeach Sarah because silence in this context is inconsistent with her testimony of self-defense, since it would be natural for Sarah to have told the police about what happened when asked earlier. b. Yes, because silence in the face of police questioning is always admissible to prove guilt. c. No, because silence can never be considered "inconsistent" with trial testimony. d. No, because what Sarah said or did not say to the police is beyond the scope of the direct testimony, and so the prosecutor cannot ask about it on cross-examination.

a. is correct. If an individual remained silent during a period of time when she would reasonably be expected to make a statement, under Rule 613 that silence can be interpreted as "inconsistent" with her current testimony that she was the one who killed the victim. b. is incorrect. Silence in the face of police questioning is not always admissible to prove guilt, and at any rate this information is being offered to impeach Sarah, not to prove that she in fact was guilty. c. is incorrect. If an individual remained silent during a period of time when she would reasonably be expected to make a statement, under Rule 613 that silence can be interpreted as "inconsistent" with her current testimony that she was the one who killed the victim. d. is incorrect. This is impeachment evidence, and impeachment evidence is never beyond the scope under Rule 611.

Oliver is an eyewitness to an armed robbery, and at trial he testifies for the prosecutor. On direct he testifies that he was about to enter his bank when he saw the defendant running out of the bank carrying a gun. He identifies the defendant in court. On cross-examination, the defense attorney asks Oliver if he ever filed a false insurance claim for his home insurance. This question is: a. Admissible, as long as the defense attorney has a good faith basis for believing that Oliver has filed a false insurance claim. b. Admissible, but only if the defense attorney has a good faith basis for believing that Oliver has filed a false claim at some point and if the defense attorney gave advance notice to the prosecutor that he was going to ask the question. c. Inadmissible, because it is beyond the scope of Oliver's direct testimony. d. Inadmissible, because it is irrelevant to whether the defendant committed the crime.

a. is correct. Rule 608(b) allows an opposing party to impeach a witness with a specific act of a prior dishonest action as long as the opposing party has a good faith belief that the act occurred. There is no requirement of notice. The question is not beyond the scope of Oliver's direct examination because impeachment is never beyond the scope under Rule 611. And the question is not irrelevant because a witness' credibility is always relevant. b. is incorrect. There is no notice requirement for Rule 608(b). c. is incorrect. The question is not beyond the scope of Oliver's direct examination because impeachment is never beyond the scope under Rule 611. d. is incorrect. The question is not irrelevant because a witness' credibility is always relevant.

Theresa is on trial for aggravated assault. She has testified in her defense. She has three prior convictions: one for aggravated assault (a felony) from six years ago, one for fraudulently filing a false police report (a misdemeanor) three years ago, and one for selling narcotics (a felony) one year ago. The prosecutor seeks to admit all three prior convictions as evidence in order to impeach her through proof of character for dishonesty. Which convictions (if any) are admissible? a. The aggravated assault is almost certainly inadmissible; filing a false police report is certainly admissible, and the selling of narcotics will be admissible if the probative value of the evidence in proving the defendant's propensity for dishonesty outweighs its prejudicial effect on the defendant. b. The aggravated assault is almost certainly inadmissible; filing a false police report is certainly admissible, and the selling of narcotics will be admissible if the probative value of the evidence in proving the defendant's propensity for committing crimes outweighs its prejudicial effect on the defendant. c. The aggravated assault is almost certainly admissible; filing a false police report is admissible if the probative value of the evidence in proving the defendant's propensity for dishonesty outweighs its prejudicial effect on the defendant, and the narcotics sale is almost certainly inadmissible. d. The aggravated assault is almost certainly admissible; filing a false police report is admissible if the probative value of the evidence in proving the defendant's propensity for dishonesty outweighs its unfair prejudice to the defendant, and the narcotics sale will be admissible if the probative value of the evidence in proving the defendant's propensity for committing crimes outweighs its prejudicial effect on the defendant.

a. is correct. Rule 609 governs the admissibility of prior convictions when used to impeach witnesses. Felonies which are not crimes of falsity (such as the aggravated assault and the narcotics sales) are admissible if the probative value of the evidence in proving the defendant's propensity for dishonesty outweighs its unfair prejudice to the defendant. In the case of the aggravated assault, the prior conviction is almost certainly inadmissible, because its similarity to the current crime makes it extremely unfairly prejudicial to the defendant. The filing of a false police report is a crime of falsity, so it is automatically admissible under Rule 609(a)(2). b. is incorrect. Rule 609 governs the admissibility of prior convictions when used to impeach witnesses. The probative value that is measured under Rule 609, however, is only the probative value to prove propensity for truthfulness, not propensity for criminal behavior. Thus, the probative value for committing crimes cannot be part of the balancing test. c. is incorrect. Rule 609 governs the admissibility of prior convictions when used to impeach witnesses. The rule for felonies against criminal defendants is that the probative value to prove propensity for dishonesty must outweigh the unfair prejudice to the defendant. The prior conviction for aggravated assault is identical to the crime for which the defendant is now on trial, so the unfair prejudice would be extremely high, while the probative value to prove propensity for dishonesty is very low for a crime of violence, so the prior conviction for aggravated assault is almost certainly not admissible. d. is incorrect. Rule 609 governs the admissibility of prior convictions when used to impeach witnesses. The rule for felonies against criminal defendants is that the probative value to prove propensity for dishonesty must outweigh the unfair prejudice to the defendant. The prior conviction for aggravated assault is identical to the crime for which the defendant is now on trial, so the unfair prejudice would be extremely high, while the probative value to prove propensity for dishonesty is very low for a crime of violence, so the prior conviction for aggravated assault is almost certainly not admissible.

Winslow Laboratories is suing Hanover, Inc. for patent infringement. As part of the discovery process, Winslow deposed five of Hanover's employees, including Hanover's Vice-President in charge of Research and Development. Later, the Vice-President also submitted a sworn affidavit relating to the case. At trial, Hanover decided not to call the Vice-President as a witness because Hanover's lawyers were worried that he would not seem credible to the jury. Instead, Hanover sought to admit the Vice-President's deposition testimony and his affidavit. The evidence is being offered for the truth of the matter asserted. Winslow objects to this as hearsay. Should the judge admit or preclude the Vice-President's deposition testimony and affidavit? a. The deposition and affidavit are hearsay and should be precluded. b. The deposition is admissible, but the affidavit should be precluded. c. The affidavit is admissible, but the deposition should be precluded. d. Both the deposition and the affidavit are admissible.

a. is correct. Rule 804(b)(1) provides for an exception for prior testimony which was given under oath with an opportunity for cross-examination, but the rule only applies if the declarant is now unavailable. In this case, the declarant (the Vice-President) is available, but the defendant simply chose not to call him to the stand. There is no other hearsay exception which will allow this testimony. b. is incorrect. Rule 804(b)(1) provides for an exception for prior testimony which was given under oath with an opportunity for cross-examination, but neither the deposition nor the affidavit meet all of the requirements of 804(b)(1). c. is incorrect. Rule 804(b)(1) provides for an exception for prior testimony which was given under oath with an opportunity for cross-examination, but neither the deposition nor the affidavit meet all of the requirements of 804(b)(1). d. is incorrect. Rule 804(b)(1) provides for an exception for prior testimony which was given under oath with an opportunity for cross-examination, but neither the deposition nor the affidavit meet all of the requirements of 804(b)(1)

Garrett's dog bit Kyle in the leg. Kyle sued Garrett, seeking monetary damages. Under the applicable law, every dog owner is strictly liable for any injuries caused by his dog. At trial, Kyle offers evidence that Garrett's dog bit three other people in the year before biting Kyle. In his defense, Garrett offers evidence that Kyle kicked the dog just before the dog bit Kyle. Is either piece of evidence admissible? a. No, because both pieces of evidence are irrelevant. b. The evidence that Garrett's dog bit three other people is inadmissible as propensity evidence, but the evidence that Kyle kicked the dog is admissible. c. The evidence that Kyle kicked the dog is inadmissible because it is unfairly prejudicial, but the evidence that Garrett's dog bit three other people is admissible because it shows that Garrett had knowledge that his dog was dangerous. d. Both pieces of evidence are admissible.

a. is correct. Since the law is strict liability, it is legally irrelevant whether the dog was known to be dangerous or whether Kyle provoked the dog. b. is incorrect. The evidence that Garrett's dog bit three other people is not propensity evidence because it also proves that Garrett knew the dog was dangerous, but it is still inadmissible on relevance grounds. The evidence that Kyle kicked the dog is inadmissible because the applicable law is strict liability, so it is irrelevant whether Kyle provoked the dog. c. is incorrect. The fact that Garrett's dog bit three other people is not admissible because Garrett is liable for the dog bite whether or not the dog is known to be dangerous. d. is incorrect. The two pieces of evidence do not prove anything that the jury needs to know in order to decide the case.

Louis is on trial for robbing a convenience store. The prosecutor seeks to call his girlfriend as a witness to testify that Louis was a regular user of heroin, and spent over $50 on heroin every day. She will also testify that Louis owed his heroin dealer over $500. Louis objects to this testimony as inadmissible character evidence. The judge should: a. Admit the testimony about Louis' heroin use and his debt to the dealer to prove motive. b. Admit the testimony about Louis' heroin use and his debt to prove that he has a propensity to commit crimes. c. Admit the testimony about Louis' heroin use and debt only after Louis calls a character witness to testify that he is law-abiding and honest. d. Preclude the testimony about Louis' heroin use and debt.

a. is correct. The evidence has significant probative value in proving that Louis had a motive to commit the crime, and it is admissible for that purpose under Rule 404(b). If Louis requests it, the judge will give a limiting instruction telling the jury not to consider the evidence for character purposes. b. is incorrect. If offered to prove propensity, the evidence is inadmissible under Rule 404(a). c. is incorrect. The evidence is not being offered to prove character, and so the prosecutor does not have to wait for Louis to open the door with his own character evidence. Even if Louis did offer character evidence, under Rule 405 the prosecutor would not be permitted to admit extrinsic evidence of specific act evidence, only opinion and reputation evidence. Thus, the prosecutor ask about Louis' heroin use when cross-examining Louis' character witness, but could not bring in extrinsic evidence to prove his propensity to commit crimes. d. is incorrect. Although the evidence is inadmissible if offered to prove propensity, it has a probative value aside from the propensity purpose.

Woodlands Construction built a building for Ace Insurance Company. There was a dispute about the final cost of the project: Ace was contractually obligated to pay the full cost of all the building supplies, but Ace claimed that Woodlands was exaggerating the cost of the building supplies in its final invoice. Woodlands sent Ace four letters over the course of a month, demanding the money that it claimed was still owed and detailing the amount that it paid for the each of the supplies, but Ace refused to pay the disputed amount. Finally Woodlands sued Ace. At trial, Woodlands sought to introduce the letters it sent to Ace which described the amount it paid for each of the building supplies. Ace objected to these letters as hearsay. Should the court admit or preclude the letters? a. Preclude the letters as hearsay. b. Admit the letters as a business record. c. Admit the letters as recorded recollection. d. Admit the letters as a party-opponent statement.

a. is correct. The letters are out-of-court statements offered to prove the cost of the building supplies. The letters fail the requirements of Rule 803(6)--they are not records that are kept in the regular course of business, and it is not a regular practice to make this kind of record. Also, the circumstances indicate a lack of trustworthiness. Rule 803(5) only applies when a witness is on the stand and has forgotten certain facts, and so uses his or her own record to get the information to the jury. There is no evidence that such a foundation has been laid here. And Woodlands is offering its own letters; thus these are not statements made by a party opponent, as required by Rule 801(d)(2). b. is incorrect. The letters fail the requirements of Rule 803(6)--they are not records that are kept in the regular course of business, and it is not a regular practice to make this kind of record. Also, the circumstances indicate a lack of trustworthiness. c. is incorrect. Rule 803(5) only applies when a witness is on the stand and has forgotten certain facts, and so uses his or her own record to get the information to the jury. There is no evidence that such a foundation has been laid here. d. is incorrect. Woodlands is offering its own letters; thus these are not statements made by a party opponent, as required by Rule 801(d)(2).

Which of the following pieces of evidence is irrelevant under Rule 401? a. During a political rally protesting police treatment of racial minorities, Donald allegedly broke a window of the police headquarters. Donald is now being prosecuted for criminal damaging. The elements of criminal damaging are: recklessly or knowingly causing damage to state property without permission or authority. Donald is Black. In Donald's defense, he wishes to present statistical evidence that the police use excessive force when arresting Black suspects at a rate three times more often than when they arrest white suspects. b. Stacy allegedly called Joshua on the telephone, used an anti-Semitic slur against him, and threatened to kill him and his family. She is charged with aggravated telephone harassment. The elements of aggravated telephone harassment are: knowingly communicating with another by telephone and knowingly causing fear of injury by reason of the victim's race, ethnicity, or religion. The prosecutor wishes to present evidence that Joshua, the victim, is Jewish. c. Bernard is charged with embezzling hundreds of thousands of dollars from his company. He was arrested after a three-week investigation, and during that time, he and his lawyer went to the prosecutor's office twice to attempt to explain his conduct. The prosecutor wishes to introduce evidence that the day before Bernard was arrested, he went to his office on a Sunday and shredded every document in his file cabinet. d. Debra is charged with driving a vehicle while under the influence of alcohol after she ran her car off the road and crashed into a tree. At the scene of the accident, she refused to take a Breathalyzer test. However, she was taken to the hospital and blood was drawn from her without her consent while she was being treated for her injuries. The prosecutor wishes to introduce evidence that the blood—which was drawn from her body approximately one hour after the accident—showed a blood-alcohol level of .15, well above the legal limit of .08.

a. is correct. The only relevant issues in the case are whether Donald caused the damage, whether he did so knowingly or recklessly, and whether he had permission. Whether he believed his actions were justified in order to make a political point is irrelevant to determining his guilt. b. is incorrect. Ordinarily an individual's religious beliefs would be irrelevant to a case—and they are inadmissible under Rule 610 if offered to impeach a witness—but in this case the prosecutor must prove that the defendant actually committed this crime with a discriminatory purpose. This fact is more likely to be true if the victim is in fact a member of the religion that the defendant is discriminating against. c. is incorrect. Bernard was obviously aware that the prosecutor was considering charges against him, and thus his shredding of the documents is relevant to prove he had consciousness of guilt—that is, that he believed that the documents would somehow incriminate him. From his consciousness of guilt, a reasonable jury could infer that Bernard was, in fact, guilty. d. is incorrect. The evidence that Debra's blood alcohol level was so high only an hour after the crash is directly relevant to whether she was drunk when she crashed the car, because it makes the fact that she was drunk at the time of the accident more likely than it would be without the evidence. The fact that she did not consent to the blood test might mean the blood is excluded on constitutional grounds, but it does not affect its relevance under the rules of evidence.

Julie Ann came home very late one night, and she was drunk, crying, and very upset. Her boyfriend Paul confronted her as soon as she got home, demanding to know where she had been. She started crying even harder, and explained that she was at a bar with some old college friends, and one of them, a man named Kyle, offered to drive her home at the end of the night. She agreed to go with him, but instead of taking her home, he drove her to an empty parking lot and raped her in his car. She said that he then drove her back to the bar and told her to get out of the car. She then walked ten blocks home. Kyle was arrested and charged with rape. At his trial, Julie Ann testified for the prosecution, and her story was consistent with what she told her boyfriend Paul on that night. The prosecutor then called Paul to the stand to report what Julie Ann told him when she got home. The defense attorney objects. Paul's testimony is: a. Admissible under the excited utterance exception to the hearsay rule. b. Admissible as a present sense impression exception to the hearsay rule. c. Admissible because the witness has already directly testified to these facts. d. Inadmissible because it is hearsay.

a. is correct. The testimony is hearsay, because it is an out-of-court statement offered for the truth of the matter asserted (to prove that Julie Ann was raped by Kyle). However, at the time Julie Ann made the statement, she was "crying and upset," and so therefore in an excited state of mind, and the excitement was related to the subject of her statement (the rape). b. is incorrect. The testimony is hearsay, because it is an out-of-court statement offered for the truth of the matter asserted (to prove that Julie Ann was raped by Kyle). It is not admissible under the present sense impression exception because the statement was not made during or immediately after the event (Julie Ann was driven back to the bar and then walked ten blocks home, so we can assume that at least ten minutes passed between the rape and her statement). c. is incorrect. The testimony is hearsay, because it is an out-of-court statement offered for the truth of the matter asserted (to prove that Julie Ann was raped by Kyle). A witness' prior statement is not automatically admissible unless—pursuant to 801(d)(1)—it is inconsistent with her prior testimony, it is consistent and used to rehabilitate the witness' credibility, or it is a statement of identification. None of these apply here. d. is incorrect. The testimony is hearsay, because it is an out-of-court statement offered for the truth of the matter asserted (to prove that Julie Ann was raped by Kyle). Although too much time had passed between the event and the statement for the present sense impression exception to apply, the excited utterance exception will still apply when (as here) the declarant is still in an excited state of mind at the time she made the statement.

An oil tanker belonging to Davis Oil ruptured, and the resulting oil spill damaged the coastline of Florida and Georgia. The Environmental Protection Agency ("EPA") sent a team out to investigate the extent of the damage. The EPA team interviewed fishermen and coastal residents, and issued a report. Local businesses and fishermen are suing Davis Oil in a class action suit. They seek to admit parts of the EPA report as evidence against Davis. Which parts of the report (if any) are admissible? a. The conclusion of the report, which states that "the damage to the local economy exceeds $300 million, and Davis Oil did nothing to mitigate this damage during the first three days after the oil spill." b. A statement made by Frank Waters, a fisherman from Georgia, who is quoted in the report as having said: "I was out on the water for three days after the oil spill, and I didn't see any Davis Oil personnel anywhere helping to contain the spread of the oil." Frank Waters is not a party to this case. c. A statement made by Steven Haggerty, a fisherman from Florida, who is quoted in the report as having said: "I was near the Davis tanker when it ruptured, and I could see the crew of the boat simply abandoned their ship in lifeboats without taking any measures to stem the flow of oil." Haggerty is one of the plaintiffs in the lawsuit. d. All of the above.

a. is correct. This is admissible as a public record under Rule 803(8)(A)(iii). It sets out factual findings as a result of a legally authorized investigation of a public office. b. is incorrect. Although the report itself is a public record and therefore admissible under Rule 803(8)(A), any third party statement inside the report is hearsay-within-hearsay, and there is no exception for these statements. c. is incorrect. Although the report itself is a public record and therefore admissible under Rule 803(8)(A), any third party statement inside the report is hearsay-within-hearsay, and there is no exception for these statements. Although Haggerty is a party in this case, a party cannot offer his or her own statement as an "admission." d. is incorrect. Although the factual findings of the public record are admissible under Rule 803(8)(A)(iii), the direct quotes inside the record are hearsay within hearsay, with no exception.

Rick owned a home near a river on a flood plain, and one year after many days of rain, the river rose and flooded his house, causing severe damage. Rick contacted his insurance company, but the insurance company refused to pay anything for the damage. Rick ultimately sued the insurance company for the damages he suffered. The insurance contract that Rick and the company signed was ambiguous as to whether the policy covered flood damage for houses inside the flood plain. Under the relevant contract law, the oral representations of the parties that were made when signing the contract are relevant when a court is interpreting the meaning of the contractual provisions. Rick testified that the insurance agent never told him that flood damage was not covered. The defendant insurance company called the agent who sold Rick the policy. The agent testified that he honestly did not remember what he said when he sold Rick the policy, since he sold the policy ten years ago. The agent will testify that every time he sells a policy to a homeowner whose home is in the flood plain, he notifies the homeowner that the basic policy does not cover flood damage. He estimates that he has sold hundreds of these policies during his career as an insurance salesman. Rick objects to this testimony. Is the insurance agent's testimony admissible? a. The insurance agent's testimony is admissible as habit evidence. b. The insurance agent's testimony is inadmissible because he has no personal knowledge of his conversation with Rick. c. The insurance agent's testimony is inadmissible because it is hearsay. d. The insurance agent's testimony is inadmissible because it requires corroboration.

a. is correct. Under Rule 406, evidence of the routine practice of an individual is admissible as habit evidence if it is specific conduct that is routinely done in response to specific situations. Here, the insurance agent testifies that he "always" warns homeowners about the flood plain exemption, and this is very specific conduct. b. is incorrect. Although the agent does not have knowledge of this specific conversation, under Rule 406 he is able to testify that he has a habit of making certain statements in certain situations. c. is incorrect. The statements are not hearsay because they are not being offered to prove the truth of the matter asserted; but to prove notice. The agent wants to prove that he said these words to Rick and that Rick heard them. d. is incorrect. There is no requirement that this testimony be corroborated; the agent could be lying about these facts, but he can be tested on cross-examination.

Greg is suing Veronica for breach of contract. Veronica has two prior convictions. Twelve years ago she was convicted of income tax fraud, a felony, and she served three years in prison. Six years ago she was convicted of filing a false police report, a misdemeanor, and she served six months in prison. Greg would like to admit the two prior convictions as evidence. Are they admissible? a. If Veronica testifies, they are both automatically admissible. b. If Veronica testifies, the filing a false police report is automatically admissible, and the income tax fraud is admissible if its probative value to impeach Veronica substantially outweighs its unfair prejudice. c. If Veronica testifies, the income tax fraud is admissible if its probative value to impeach Veronica substantially outweighs its unfair prejudice, and the filing a false police report is not admissible. d. Neither conviction is admissible whether Veronica testifies or not.

a. is correct. Under Rule 609(a)(2) any conviction for a crime of falsity that occurred within ten years is automatically admissible. The ten years is calculated from the time of "conviction, or release from confinement, whichever is later." Thus, the ten year time limit does not count any time spent in prison, so for the purposes of Rule 609, the income tax fraud conviction occurred nine years ago. b. is incorrect. Although it may appear that the income tax fraud conviction occurred over ten years ago, the time calculation does not include any time spent in prison, so for the purposes of Rule 609, the income tax fraud conviction occurred nine years ago. c. is incorrect. Although it may appear that the income tax fraud conviction occurred over ten years ago, the time calculation does not include any time spent in prison, so for the purposes of Rule 609, the income tax fraud conviction occurred nine years ago. Also, although the filing a false police report is a misdemeanor, it is a crime of falsity under Rule 609(a)(2). d. is incorrect. Under Rule 609, if Veronica testifies, both convictions may be admissible to impeach Veronica by attacking her character for truthfulness.

The plaintiff is suing the defendant for personal injuries suffered when she was struck by the defendant's car. The plaintiff testified regarding the extent of her injuries. The defendant seeks to testify that when she visited the plaintiff in the hospital, the plaintiff said that if the defendant paid all of her medical bills and gave her $25,000, she would not institute legal proceedings. The plaintiff's attorney objects to the offered testimony. How should the judge rule on the admissibility of the testimony? a. inadmissible, because it is hearsay b. inadmissible, because it was an offer of compromise. c. admissible, because it is a statement against interest and, thus, an exception to the hearsay rule. d. admissible, because it is relevant to show that the plaintiff tried to blackmail the defendant, and does not really have a claim.

b. is correct. The judge should rule the testimony inadmissible. Evidence of compromises or offers to compromise is inadmissible to prove or disprove the validity or amount of a disputed claim. (A) is incorrect; the evidence is not hearsay because it is a statement made by and offered against a party. (C) is incorrect because the statement against interest exception requires that the witness be unavailable. Here, the plaintiff is available to testify. (D) may be relevant, but the facts show that it was merely an offer to compromise a disputed claim, and therefore public policy encouraging settlements precludes the introduction of such evidence. Thus, (B) is correct.

A defendant is on trial for allegedly burning down her business establishment because it was losing money. Before the defendant takes the stand in her defense, the prosecution seeks to introduce testimony from an insurance agent that the defendant purchased two insurance policies for the building within a month before the fire. Each policy had been purchased from a different insurance carrier and each policy was in the amount of the full value of the business. The defendant's attorney objects to the introduction of this testimony. How should the court rule on the admissibility of the testimony? a. admissible only for purposes of impeachment when the defendant takes the stand in her own defense. b. admissible as substantive evidence against the defendant. c. inadmissible, because the federal rules ban using evidence that a party carried insurance to prove that the party acted wrongfully. d. inadmissible, because the policies themselves are required to be introduced under the original document rule.

b. is correct. The testimony is admissible to show that the defendant had a motive to destroy the building. Federal Rule 411, which prohibits the admission of evidence of liability insurance to show a person acted negligently or wrongfully, does not apply to these circumstances. The insurance at issue in this case is not liability insurance; it is casualty insurance. The rationale for the exclusion of evidence of liability insurance (i.e., that a trier of fact might improperly infer that a person acted more carelessly because she knew she was insured) has no application here. Whether a person was negligent is not at issue. The insurance policies are relevant because they make a fact in issue, that the defendant set her building on fire, more likely than it would be without evidence of the policies. Because it is relevant and not subject to any exclusionary rule, the testimony regarding the insurance policies is admissible to show motive for the arson. (A) is wrong because evidence of the insurance policies is admissible as substantive evidence in the prosecution's case in chief; it is not limited to impeachment uses. The rule stated in (A) is the one applicable to character type evidence that may be brought in to impeach a criminal defendant's credibility if she takes the stand. The evidence of the insurance policies, apart from having substantive value in proving the case, is of no help in attacking the defendant's veracity when she takes the stand. Thus, this rule is totally inapplicable to this evidence. (C) is wrong because, as noted above, Rule 411 bans the use of evidence of liability insurance only. Liability insurance is not at issue here. (D) is wrong because the admission of testimony regarding the insurance policies does not violate the original document rule (also known as the best evidence rule). The rule requires the original writing to be produced to prove the terms of the writing, where the terms are material. Here, the testimony is not being sought to prove the terms of the policies, but rather that the defendant obtained them. That fact can be established independent of the policy itself.

Jane Young, a six-year-old girl, was given antibiotics for a minor infection. Unfortunately, she was allergic to the antibiotics and she became very ill, spending over two weeks in the hospital. After Jane got ill, the doctor met with his two nurses and told them that they needed to conduct a basic allergy test on every new patient with an infection before the doctor could determine which antibiotics should be prescribed. Jane's parents sued the doctor who prescribed the antibiotics, alleging that he should have known that Jane was allergic to this specific strain of antibiotics and thus should not have prescribed them. The Young family had health insurance, so they did not have to pay any expenses aside from a $100 deductible for the hospital stay. However, they sued the doctor seeking medical expenses for the hospital stay (which would ultimately go to reimburse the insurance company, except for the $100 deductible) and for mental suffering (which the Young family would keep for themselves). At trial, Daniel Young, Jane's father, testified about his mental suffering, saying that there were many days when he did not know whether his daughter would live or die, and that he had to seek therapy in order to recover from the trauma. Daniel also testified that he suffered further mental anguish because "Hospitals are so expensive, and I would lie awake at night wondering how we were going to pay for her care and whether we would be able to afford the treatment she needed." Which of the following evidence is admissible? a. In their case-in-chief, the Young family seeks to admit evidence that the doctor told his nurses to conduct allergy tests on all new patients with infections. b. In his case-in-chief, the doctor defendant seeks to admit evidence that the Young family was fully insured for the hospital stay and only paid a $100 deductible. c. Neither (A) nor (B) are admissible. d. Both (A) and (B) are admissible.

b. is correct. Although Rule 411 precludes evidence of insurance to prove negligence or wrongful conduct, the evidence of insurance in this context is being offered to prove that the Young family did not suffer additional mental trauma because they were worried that they would be unable to pay the medical expenses. In other words, Daniel Young "opened the door" to the evidence of the insurance being admissible when she claimed that she was worried about paying the medical expenses. a. is incorrect. The doctor's new policy to test all new patients for allergies is a subsequent remedial measure, and so it is inadmissible to prove liability. Since the doctor has not contested feasibility or ownership or control, there is no other relevant purpose for this evidence. c. is incorrect. Although Rule 411 precludes evidence of insurance to prove negligence or wrongful conduct, the evidence of insurance in this context is not being offered to prove negligence or wrongful conduct. d. is incorrect. The doctor's new policy to test all new patients for allergies is a subsequent remedial measure, and so it is inadmissible to prove liability. Since the doctor has not contested feasibility or ownership or control, there is no other purpose to this evidence.

Sandra and Bill made an oral contract in which Bill agreed to buy 12 different rare baseball cards from Sandra. The day after the two of them allegedly made the deal, Bill wrote down from memory the 12 different cards he had agreed to purchase as well as the purchase price for each card. The total purchase price according to his document was $5,280. The next day, Sandra delivered the cards and demanded $8,560. Bill protested, saying that was more than the price they had agreed on. Ultimately the two of them could not agree, and Sandra sued Bill for breach of contract. At trial, Bill testified about the deal he made with Sandra, but he could not remember exactly how much he had agreed to pay for each card. His attorney asked if he wrote down the prices for the cards. Bill said he did, and he confirmed that at the time he wrote down the prices, the prices were fresh in his memory and the record accurately reflected his memory at the time. His attorney then gave him the document he made the day after the deal was made, and (over Sandra's objection) Bill read it out to the jury. Bill's attorney then admitted the document into evidence (again over Sandra's objection). While the jury was deliberating, they asked to see the document, and (again over Sandra's objection) the document was sent back into the jury room for the jury to examine. Did the trial judge make the proper rulings? a. Yes, the document was properly admitted into evidence and it was appropriate to give it to the jury. b. No. Although it was correct to allow Bill to read the document to the jury, the document should not have been admitted into evidence or given to the jury. c. No. Bill should have looked at the document, then given it back to the attorney, and then testified from his own refreshed recollection. Also, the document should not have been admitted into evidence or given to the jury. d. No. Bill should not have been allowed to look at the document at all.

b. is correct. Bill was allowed to read the document to the jury under Rule 803(5) as a recorded recollection, but the document was not admitted into evidence and so cannot go back into the jury room. a. is incorrect. Bill was allowed to read the document to the jury under Rule 803(5) as a recorded recollection, but the document is not admitted into evidence and so cannot go back into the jury room. c. is incorrect. Since the document qualifies as a recorded recollection under Rule 803(5), Bill may read the document to the jury. Since Bill never testified that the document would refresh his recollection, it is not being used to refresh his recollection under Rule 612. d. is incorrect. Since the document qualifies as a recorded recollection under Rule 803(5), Bill may read the document to the jury.

Frank and Allen ran a small software company together, and they developed a scheme to cheat on their taxes. They set up a dummy corporation, Alabaster, Inc., and they then made payments to Alabaster for "consulting" work. They deducted these payments from the reported income of their actual company and then paid taxes on the fraudulently lowered income. They then reported false "losses" for Alabaster so that Alabaster would have no income. Frank and Allen were ultimately arrested and their trial was severed. In Frank's trial, the prosecutor sought to admit an e-mail written by Allen to Frank which said: "I made another $50,000 payment to Alabaster today. That brings the total payments this year to $250,000." Frank objects to the e-mail as hearsay. Is Allen's statement admissible? a. Allen's statement is admissible to prove Frank knew about the payments, but not for the truth of the matter asserted. b. Allen's statement is admissible for the truth of the matter asserted as a party-opponent statement. c. Allen's statement is admissible only if Frank is unavailable. d. Allen's statement is inadmissible for any purpose.

b. is correct. Frank and Allen were co-conspirators, and Allen's statement was made during the conspiracy and in furtherance of the conspiracy. Thus, under Rule 801(d)(2)(E), these statements are admissible as party-opponent statements. A statement made in furtherance of the conspiracy is not testimonial, since the declarant did not reasonably expect it would be used at trial, so there is no Crawford problem with admitting this statement. a. is incorrect. The statement is admissible under one of the hearsay exceptions. c. is incorrect. The statement's admissibility does not depend on the declarant being unavailable. Unavailability is required for the statement against interest exception under 804(b)(3), but this is not a statement against interest because it was not against the declarant's interest at the time he made it. d. is incorrect. The statement is admissible under one of the hearsay exceptions.

Debra is on trial for embezzlement after allegedly transferring six thousand dollars from a company account into her own personal account. The prosecutor has evidence that two years earlier, Debra stole two hundred dollars from her last employer by endorsing a check that was meant for the employer and depositing it into her own account. Will the prosecutor be able to admit this evidence? a. Yes, the prosecutor can admit this evidence in her case in chief under Rule 404(b). b. Yes. If Debra calls a character witness who testifies that Debra has an honest character and is not the type of person who would steal, then the prosecutor can ask the character witness about the prior embezzlement on cross-examination of the character witness. c. Yes. If Debra calls a character witness who testifies that Debra has an honest character and is not the type of person who would steal, then the prosecutor can ask the character witness about the prior embezzlement on cross-examination of the character witness and/or the prosecutor can offer evidence of the prior embezzlement in her rebuttal case. d. No, the evidence of Debra's prior embezzlement cannot be admitted at trial.

b. is correct. If Debra calls a character witness who testifies as to Debra's honest character, Rule 405 allows the prosecutor to impeach that witness by asking her questions on cross-examination about Debra's specific acts if they are contrary to the character trait that the witness described. a. is incorrect. There is no 404(b) purpose to this evidence; that is, the only probative value of this evidence is to prove that Debra has a propensity to steal money. Under Rule 404(a), the prosecutor cannot be the first to offer propensity evidence in a criminal case. c. is incorrect. Rule 405 does not allow either party to prove propensity evidence with specific act evidence on direct, so the prosecutor could not offer evidence of the prior embezzlement in her rebuttal case. d. is incorrect. Although generally Rule 405 does not allow specific act evidence to prove propensity, it does allow specific act evidence under certain circumstances.

George is on trial for assault. The prosecutor argues that George got in a fight with a man outside a bar and beat him severely. George testifies in his own defense, and tells the jury that he was not present at the bar; he was across town at home eating dinner at the time the beating occurred. George then calls a character witness. Which of the following statements by the character witness would most likely be admissible? a. "I have known George for ten years and in my opinion he is an honest and trustworthy person who never lies." b. "I have known George for ten years and in my opinion he would never be violent against anyone." c. "Last year I was with George when a man came up to him and tried to pick a fight. The man was insulting George, he called George a racist epithet, and even threw George to the ground. But George just got up and walked away; he never got angry or raised his fists." d. "George is a very generous man; he gives thousands of dollars to charity every year and volunteers his time at the local homeless shelter."

b. is correct. Rule 404(a) allows a defendant to call a character witness to testify about a pertinent character trait of the defendant. Whether George has a violent propensity is a pertinent character trait. a. is incorrect. Although Rule 608(a) allows a character witness to testify about another witness' propensity for credibility, the character witness cannot testify about how honest a witness is until after the witness' propensity for credibility has been attacked. There is no indication here that the prosecutor attacked George's propensity for credibility. c. is incorrect. Although Rule 404(a) allows a defendant to call a character witness to testify about his opinion about a pertinent character trait of the defendant, Rule 405 limits this testimony to opinion or reputation evidence. This evidence pertains to a specific act, and therefore is not admissible on direct examination. d. is incorrect. Rule 404(a) allows a defendant to call a character witness to testify about his opinion about a pertinent character trait of the defendant, but George's generosity and gifts to charity are not relevant to whether he committed an assault.

Detective Starr works as a narcotics detective for the Jonesville Police Department. He has been indicted on charges of theft and cocaine possession with intent to distribute. The prosecutor alleges that six months ago Detective Starr arrested a suspect for cocaine possession, and found five kilograms of cocaine in the trunk of the suspect's car, but only turned in four kilograms to the police property room as evidence, keeping one kilo for himself in order to sell. Detective Starr denies the charges, arguing that there were only four kilograms of cocaine in the trunk originally and that he never stole any cocaine. At trial, Starr calls two character witnesses. The first is Detective Reynoso, Starr's partner. Detective Reynoso will testify that he has worked with Starr for five years and that Starr is the bravest police officer he has ever known. The second witness is Sarah, Detective Starr's girlfriend. Sarah will testify that she has known Starr for ten years and that he is extremely honest and law-abiding. The prosecutor objects to both witnesses. How should the court rule? a. Admit Detective Reynoso's testimony but not Sarah's testimony. b. Admit Sarah's testimony but not Detective Reynoso's testimony. c. Admit both Detective Reynoso's testimony and Sarah's testimony. d. Preclude both Detective Reynoso's testimony and Sarah's testimony.

b. is correct. Rule 404(a)(2) allows a criminal defendant to admit propensity evidence for any "pertinent trait," and Sarah's testimony that Starr is honest and law-abiding tends to prove that he did not commit the crime of stealing the cocaine with the intent to sell it. However, Detective Reynoso's testimony that Detective Starr is a brave a police officer is not pertinent to whether Starr stole the cocaine or possessed it with the intent to sell it. a. is incorrect. Although Rule 404(a)(2) allows a criminal defendant to admit propensity evidence in his defense, the evidence must be about a "pertinent trait." Detective Starr's bravery as a police officer is not pertinent to whether he stole the cocaine or possessed it with the intent to sell it. c. is incorrect. Although Rule 404(a)(2) allows a criminal defendant to admit propensity evidence in his defense, the evidence must be about a "pertinent trait." Detective Starr's bravery as a police officer is not pertinent to whether he stole the cocaine or possessed it with the intent to sell it. d. is incorrect. Rule 404(a)(2) allows a criminal defendant to admit propensity evidence for any "pertinent trait," and Sarah's testimony that Starr is honest and law-abiding tends to prove that he did not commit the crime of stealing the cocaine or possessed it with the intent to sell it.

Chrome Motor Company developed a new type of SUV called the Growler. Unfortunately, within two weeks after the Growler was out on the market, three different owners reported problems with the accelerator pedal. In all three cases, the driver reported that the accelerator pedal stuck, causing the car to accelerate uncontrollably, resulting in an accident that caused injury and property damages. All three owners sued Chrome, alleging that the Growler had an unsafe design that caused the accelerator to stick. Chrome settled with the first plaintiff for $50,000 and the second plaintiff for $100,000. However, the third plaintiff rejected all settlement offers and took the case to trial. At trial, the attorney for the third plaintiff sought to admit the fact that Chrome settled the other two cases, as well as the amount of money Chrome paid for the settlement. The attorney argued that these settlements proved that Chrome acknowledged the design was faulty. Chrome objected to this evidence. Should the judge admit or preclude the evidence of the settlements? A. Admit the settlement that was paid to the second plaintiff, but not the first plaintiff. B. Preclude any evidence from either settlement. C. Allow evidence of the fact that Chrome settled the other two cases, but do not allow evidence of the amount of money that was paid. D. Allow evidence of the fact that Chrome settled the other two cases, and allow evidence of the amount of money that was paid.

b. is correct. Rule 408 precludes any evidence of a settlement in order to prove liability, and there is no other reason that these settlements are relevant. Rule 408 applies to completed settlements as well as settlement offers. a. is incorrect. here is no legal reason to distinguish between the two settlements. c. is incorrect. It is true that the amount of money Chrome paid has a low probative value and probably a high amount of unfair prejudice. However, the fact that Chrome agreed to settle other cases is also barred by Rule 408 because it would discourage any defendant from settling a case in a multi-party litigation. d. is incorrect. Rule 408 applies to completed settlements as well as settlement offers.

George was loading a truck at work with boxes when the forklift operator accidentally hit him with the forklift, crushing his torso and breaking six ribs. George was rushed to the hospital and prepped for surgery to repair damage to his internal organs. As he was waiting to go into surgery, the Vice-President of the company he worked for came to him and said: "George, this was a terrible thing to happen. I want to assure you that the company will pay for this surgery and any rehabilitation that you need." George is now suing his company for negligent training and supervising of its employees, and he wants to testify as to the Vice-President's statement. Which rule of evidence would be the BEST one for the company to cite when it objects to this evidence? a. Rule 402, which bars irrelevant evidence. b. Rule 409, which bars any offer to pay medical expenses. c. Rule 408, which bars evidence of settlement offers or negotiations. d. Rule 802, which bars hearsay evidence.

b. is correct. Rule 409 bars evidence of any offer to pay medical expenses if offered to prove liability, regardless of whether there was a disputed claim at the time of the offer. a. is incorrect. Rule 402 states that irrelevant evidence is inadmissible. Although the statement made by the Vice-President has very little probative value, it is relevant because the fact that the Vice-President offered to pay for the surgery makes it a little more likely that the company is at fault in the accident. c. is incorrect. Rule 408 bars evidence of any settlement offer, but only if there was a disputed claim at the time the offer was made. In this case, there was no claim because George had not filed any kind of formal complaint against the company, and there was no dispute because the Vice-President did not contest liability in any way. Thus, Rule 408 does not apply. d. is incorrect. Rule 802 bars hearsay evidence unless one of the many exceptions applies. The statement by the Vice-President is a party-opponent statement under 801(d)(2), since he was speaking on behalf of the company, and so the statement is exempted from the hearsay rule.

Wilbur is a defense witness in a criminal case. After Wilbur testified, defense rested and the plaintiff called Emma, who works with Wilbur in the same company. Emma testifies that she has known Wilbur for six years and worked with him on numerous projects. Emma testifies as follows: Plaintiff's Attorney: What is your opinion of Wilbur's honesty? Emma: I would never trust Wilbur. I think he lies whenever he sees a benefit to it. He is certainly the type of person who would lie on the stand if he saw something in it for him. Plaintiff's Attorney: Can you tell me why you have this opinion? Emma: Sure. Well, there are a lot of reasons. The first time we worked together on a project, his part of the project was late, and so the entire project missed its deadline by a week. I learned a few months later that Wilbur had gone to our boss afterward and told him that the delay was my fault, even though my part of the project was on time. I was furious. Is Emma's testimony admissible? a. Yes, all of the testimony is permissible character testimony. b. Only in part. Emma should be allowed to make the first general statement about Wilbur's character, but she should not be permitted to describe the episode with the delayed project. c. Only in part. Emma should be allowed to make the second statement describing the episode with the delayed project, but she should not be permitted to make the general statement about Wilbur's character. d. No, all of the testimony is impermissible character evidence.

b. is correct. Rule 608(a) allows a character witness to give opinion or reputation evidence regarding the truthfulness of a fact witness. However, Rule 608(b) states that a character witness may not give specific act evidence to attack or support a witness' character for truthfulness. a. is incorrect. Although a character witness is permitted to talk about the fact witness' character for truthfulness, Rule 608 limits the type of evidence which can be used. c. is incorrect. Rule 608(a) allows a character witness to give opinion or reputation evidence regarding the truthfulness of a fact witness. d. is incorrect. A character witness is permitted to talk about the fact witness' character for truthfulness, although Rule 608 limits the type of evidence which can be used.

Betsy is suing her doctor, arguing that the doctor did not exercise the required level of care when he operated on her knee. Her husband testifies in her case, describing the difficulty that Betsy had moving around in the weeks after the surgery. At one point during his testimony, he testifies that "two months after the doctor performed the operation, Betsy could still not walk up the stairs of our home." The doctor's attorney cross-examines the husband and asks him whether it is true that two months after the operation, he had dinner with a friend and he said: "Betsy is really recovering well after her surgery. She can go up and down the stairs at our house with no problem." Can the doctor's attorney ask her this question? a. No, because it is beyond the scope of the direct. b. Yes, but Betsy can request a limiting instruction which tells the jury to only consider the prior statement for impeachment purposes and not for the truth of the matter asserted. c. Yes, but if the husband denies having made the statement, the doctor is stuck with the answer; he cannot call the friend to testify about what the husband said. d. No, unless the doctor's attorney discloses the statement to the husband before asking her about it.

b. is correct. Rule 613 allows a party to impeach a witness with a prior inconsistent statement; this statement is inconsistent with her trial testimony. However, the statement is being offered to impeach the husband, not for the truth of the matter asserted, so a limiting instruction is required. This is not a collateral matter—that is, it is relevant to a material fact in the case (the complications that resulted from the surgery). Since it is not collateral, the doctor could call the friend to verify that the husband made the statement; he will not be stuck with the husband's answer. a. is incorrect. The statement is being offered to impeach the husband, and impeachment material is never beyond the scope. c. is incorrect. Because this is not a collateral matter, the doctor could call the friend to verify that the husband made the statement; he will not be stuck with the husband's answer. d. is incorrect. Rule 613(a) states that the impeaching attorney need not disclose a statement to the witness ahead of time before using it to impeach her.

In a civil case, Brent wants to admit a statement made by Carl as a statement against interest. Brent has already established that the statement was so contrary to Carl's pecuniary interest that a reasonable person would not have made the statement unless he believed it to be true. What else does Brent need to establish in order to admit Carl's statement under this exception? a. The opposing party had a prior opportunity to cross-examine Carl. b. Carl is now unavailable to testify. c. There are corroborating circumstances that indicate Carl's statement is trustworthy. d. All of the above.

b. is correct. Rule 804(b)(3), like all Rule 804 exceptions, requires that the declarant be unavailable to testify. a. is incorrect. Rule 804(b)(3) has no requirement that the opposing party have a prior opportunity to cross-examine Carl. c. is incorrect. In a criminal case, if the statement tends to expose the declarant to criminal liability, Rule 804(b)(3)(B) requires corroboration, but this is a civil case. d. is incorrect. Rule 804(b)(3) does not contain all of these requirements in a civil case.

Louis was stopped on the street pursuant to a legal Terry stop and the police officers frisked him and recovered a handgun. The officers ran his records and found that three years ago Louis had been convicted of burglary, a felony. Louis was charged with knowingly possessing a firearm while being a convicted felon. The elements of the crime are that the defendant (1) knowingly possessed a firearm; and (2) the defendant is a convicted felon. At trial, Louis admitted that he had been convicted of burglary three years ago, but he testified that he did not know that burglary was a felony. He will testify that he only received a six month sentence, and that he honestly believed that a crime was not a felony unless the defendant served a year or more in prison. The prosecutor objects to Louis' testimony. How should the court rule? a. Overrule the objection and allow Louis to testify. b. Sustain the objection because Louis' testimony is irrelevant. c. Sustain the objection because Louis' testimony violates Rule 404's ban on propensity evidence. d. Sustain the objection because Louis' testimony is hearsay, and no exception applies.

b. is correct. The elements of the crime are that the defendant (1) knowingly possessed a firearm; and (2) the defendant is a convicted felon. Whether the defendant knows he is a convicted felon is irrelevant; thus Louis' testimony about his lack of knowledge is irrelevant. a. is incorrect. Louis' testimony is barred by one of the rules of evidence. To understand which rule, look at what Louis is trying to prove with this testimony and look at the elements of the crime. c. is incorrect. The testimony is not being offered to prove propensity, and so does not violate Rule 404(a). d. is incorrect. Louis is not testifying to an out-of-court statement, so the hearsay rule does not apply.

Draco Chemical's pipeline burst and spilled toxic chemicals into a residential neighborhood. Two separate homeowners, Thomas and Darlene, sued Draco Chemical for damages arising from the spill. The cases were severed and proceeded to trial separately. In the first case, when Thomas was the plaintiff, Draco called its chief engineer to the stand to testify. After the first case was over, the chief engineer was hit by a car and died. The court is now trying the second case, in which Darlene is suing Draco. Draco wants to admit the testimony of its chief engineer as evidence in its defense. Is the chief engineer's testimony admissible? a. No, because Darlene never had a chance to cross-examine the chief engineer. b. Yes, but only if Thomas had a similar motive to cross-examine the chief engineer in his case as Darlene does now in her case. c. Yes, but only if the judge determines that the chief engineer's testimony is credible. d. Yes, the testimony is admissible because the engineer is now unavailable.

b. is correct. The evidence is hearsay, but it may be admissible under Rule 804(b)(1) as former testimony. For the most part, 804(b)(1) only applies if the opposing party had an opportunity to cross-examine the declarant on a prior occasion, but 804(b)(1)(B) provides that in a civil case, the former testimony is admissible if a "predecessor in interest" had an opportunity and a similar motive to cross-examine the declarant. In this case, Thomas had the opportunity to cross-examine the chief engineer, so if he also had a similar motive to cross-examine as Darlene now has (which is likely), the testimony is admissible against Darlene. a. is incorrect. The evidence is hearsay, but it may be admissible under one of the hearsay exceptions even if Darlene never had the chance to cross-examine the chief engineer. c. is incorrect. There is no hearsay exception that relies upon the trial judge's determination that a declarant's testimony is credible. The jury has the sole power to determine the credibility of a witness. d. is incorrect. The evidence may be admissible under one of the hearsay exceptions since the chief engineer is now unavailable, but Draco must prove that Thomas had a similar motive to cross-examine that Darlene does now.

Police had probable cause to believe that drugs were being stored in apartment 5D on the fifth floor of a certain building. They obtained a search warrant and searched the apartment, successfully finding a large quantity of heroin. However, there were no people in the apartment at the time, and the police were unable to determine who owned or lived in the apartment. The police then stationed themselves outside apartment 5D and waited. A teenager and his mother who lived in the building walked by and saw the police officers standing outside the apartment. The teenager turned to his mother and laughed, saying: "Those cops are morons. Big Jim isn't coming back to his place if there are a bunch of police waiting outside." A few minutes later an older woman walked by the apartment and the officer asked her: "Do you know where Big Jim lives?" and she pointed at the door of apartment 5D and then walked away. Police later arrested James Fanning for possession of heroin. At trial, the prosecutor established that in the neighborhood, Fanning is known as "Big Jim." The prosecutor then called the police officer to the stand, and asked him to testify about what the teenager said and what the older woman said. The evidence is being offered to prove that a person named "Big Jim" lived in apartment 5D. The defendant's attorney objected to this testimony as hearsay. How should the court rule? a. Admit both the statement by the teenager and the pointing by the older woman. b. Admit the statement by the teenager, but preclude the pointing by the older woman. c. Preclude the statement by the teenager, but admit the pointing by the older woman. d. Preclude both the statement by the teenager and the pointing by the older woman.

b. is correct. The statement by the teenager was non-assertive conduct; that is, the information he meant to convey was that Big Jim would not return home, not that Big Jim did in fact live in the apartment. Thus, that statement is being offered not to prove the truth of the matter being asserted (that Big Jim would not return home), but to prove a fact that can be inferred from the fact that the declarant made the statement (that Big Jim lived in the apartment). The pointing by the older woman, although not an oral statement, was still a "statement" in the sense that it was intended to communicate a fact (that Big Jim lived in that apartment), and the prosecutor wants to admit this statement to prove the fact being asserted—thus, it is a hearsay statement. a. is incorrect. The statement by the teenager was non-assertive conduct; that is, the information he meant to convey was that Big Jim would not return home, not that Big Jim did in fact live in the apartment. Thus, that statement is being offered not to prove the truth of the matter being asserted (that Big Jim would not return home), but to prove a fact that can be inferred from the fact that the declarant made the statement (that Big Jim lived in the apartment). c. is incorrect. The statement by the teenager was non-assertive conduct; that is, the information he meant to convey was that Big Jim would not return home, not that Big Jim did in fact live in the apartment. Thus, that statement is being offered not to prove the truth of the matter being asserted (that Big Jim would not return home), but to prove a fact that can be inferred from the fact that the declarant made the statement (that Big Jim lived in the apartment). The pointing by the older woman, although not an oral statement, was still a "statement" in the sense that it was intended to communicate a fact (that Big Jim lived in that apartment), and the prosecutor wants to admit this statement to prove the fact being asserted—thus, it is a hearsay statement. d. is incorrect. The statement by the teenager was non-assertive conduct; that is, the information he meant to convey was that Big Jim would not return home, not that Big Jim did in fact live in the apartment. Thus, that statement is being offered not to prove the truth of the matter being asserted (that Big Jim would not return home), but to prove a fact that can be inferred from the fact that the declarant made the statement (that Big Jim lived in the apartment). The second part of his answer is correct: the pointing by the older woman, although not an oral statement, was still a "statement" in the sense that it was intended to communicate a fact (that Big Jim lived in that apartment), and the prosecutor wants to admit this statement to prove the fact being asserted—thus, it is a hearsay statement.

Tracy is suing Windham Ladder Co., claiming that their ladder was manufactured with a design defect. She testifies on direct that the first time she used the ladder, it collapsed and she fell to the ground, breaking her ankle. Which question would most likely be permitted during cross-examination? (Assume that the opposing party has a good faith basis for believing that all of the facts being asked about are true). a. "Isn't it true that you owe thousands of dollars in credit card debt?" b. "Isn't it true that you lied on your mortgage application when you tried to buy a house last year?" c. "Isn't it true that you were fired from your job for being late to work ten times in a month?" d. "Isn't it true that you have had three children out of wedlock?"

b. is correct. This is evidence of a prior dishonest action, and it is therefore relevant to Tracy's credibility and is admissible under Rule 608(b). The opposing party is allowed to ask the question, but she is stuck with the answer—that is, she cannot prove the false statement with extrinsic evidence. a. is incorrect. This information does not bear on Tracy's credibility, and it is not relevant to the case. Rule 608 only permits questions which are relevant to a witness' credibility. c. is incorrect. This information does not bear on Tracy's credibility, and it is not relevant to the case. Rule 608 only permits questions which are relevant to a witness' credibility. d. is incorrect. This information does not bear on Tracy's credibility, and it is not relevant to the case. Rule 608 only permits questions which are relevant to a witness' credibility.

Zach is on trial for killing Penelope. He calls Randy as a witness. Randy testifies that he was once roommates with a man named Gerald, and that Gerald told Randy that Gerald had killed Penelope. The prosecutor wants to object to Randy's testimony. Zach claims that Gerald's confession was a statement against interest, and that he has already tried to subpoena Gerald to testify, but that Gerald refused to testify based on the Fifth Amendment. What is the prosecutor's best response? a. "Randy's testimony does not qualify as a statement against interest, because the statement was not against Gerald's interest at the time he made it." b. "Randy's testimony does not qualify as a statement against interest, because Zach has offered no evidence to corroborate the testimony." c. "Randy's testimony does not qualify as a statement against interest, because Gerald is not unavailable." d. "Randy's testimony qualifies as a statement against interest, but it is inadmissible because it violates the Sixth Amendment right of confrontation."

b. is correct. Under Rule 804(b)(3)(B), if a statement against interest is offered in a criminal case and tends to expose the declarant (in this case, Gerald) to criminal liability, the statement must be supported by corroborating circumstances. a. is incorrect. Gerald was admitting to a serious crime, which could expose him to criminal liability. Thus, under Rule 804(b)(3), the statement was against his interest when he made it. c. is incorrect. Gerald has claimed privilege, so he is unavailable under Rule 804(a)(1). d. is incorrect. The Sixth Amendment confrontation clause right only protects criminal defendants. This testimony is being offered by a criminal defendant against the state.

A plaintiff was injured in an automobile accident when her car was hit by a pickup truck driven by the defendant. At trial of her personal injury action, the plaintiff alleges that the defendant was driving on the wrong side of the road in excess of the posted speed limit. The defendant denies these allegations and denies liability for the accident. The plaintiff seeks to introduce evidence that the defendant has a reputation in the community for being a daredevil and for being somewhat irresponsible. In fact, the plaintiff's witness would testify that the defendant is known by all his friends as "the Menace." Is the proffered testimony admissible? a. yes, because reputation evidence is a proper method of proving character b. yes, because it is relevant c. no, it is inadmissible to show that the defendant was negligent on this occasion d. no, because the defendant did not introduce evidence of his reputation for carefulness

c. is correct The testimony of the plaintiff's witness should not be admitted to show that the defendant was negligent. Character evidence as proof of conduct in the litigated event is not admissible in a civil case unless character is directly in issue (e.g., in a defamation action). Character is not directly in issue here, and so (A) and (D) are incorrect. The defendant in a criminal, but not a civil, case can introduce evidence of good character, which can then be rebutted. (B) is incorrect because, although such evidence is clearly relevant, courts exclude this evidence because its slight probative value is outweighed by the danger of unfair prejudice, the possible distraction of the jury from the main question in issue, and the possible waste of time required by examination of collateral issues.

Three masked men robbed a convenience store, during the course of which a clerk was killed. An investigation led the police to believe the defendant was one of the robbers and they placed him under arrest. The defendant protested that he was innocent and volunteered to take a lie detector test. The test was conducted by a qualified polygraph expert. According to the expert's analysis of the test, the defendant lied about his participation in the armed robbery. At the defendant's jury trial for the armed robbery, the prosecution calls the expert to the stand to testify as to her analysis of the results of the polygraph test. The defense objects. If the objection is sustained, what is the likely reason? a. the expert's testimony would violate the defendant's right against self-incrimination if he elects not to take the stand in his own defense. b. the expert's testimony would violate the defendant's right against self-incrimination, regardless of whether he elects to take the stand in his own defense. c. polygraph evidence is considered to be unreliable and potentially confusing to jurors. d. the federal rules specifically classify polygraph evidence as irrelevant.

c. is correct. The expert's testimony should not be permitted under Federal Rule 403 because the probative value of polygraph evidence is substantially outweighed by the tendency of its results to mislead and confuse the jury. Federal Rule 702 permits opinion testimony by a qualified expert where the subject matter is one where scientific, technical, or other specialized knowledge would assist the trier of fact in understanding the evidence or determining a fact in issue. However, the methodology underlying the opinion must be reliable (i.e., the proponent must show that the opinion is based on sufficient facts or data, the opinion is the product of reliable principles and methods, and the expert has reliably applied the principles and methods to the facts of the case). Furthermore, Rule 403 gives a trial judge broad discretion to exclude relevant evidence if its probative value is substantially outweighed by the danger of unfair prejudice, confusion of the issues, or misleading the jury. Most jurisdictions have concluded that the probative value of a polygraph test is slight because of its significant rate of error. On the other side of the balancing test, the tendency of jurors to give too much weight to a polygraph test makes the danger of unfair prejudice high. Therefore, the test's unreliability and the risk of confusion from the test's results justify excluding the expert's testimony. (A) is incorrect because the defendant volunteered to take the polygraph test. The Fifth Amendment right against self-incrimination applies only when the defendant is compelled to make the statements. The fact that he did not take the stand in his own defense would not change this result because once a voluntary statement is made, it cannot be kept out of the trial on self-incrimination grounds. (B) is also incorrect because, as discussed above, the defendant volunteered to take the polygraph test. His right against self-incrimination is violated only if he is compelled to testify against himself. Just as a defendant's voluntary confession cannot be kept out of the trial on self-incrimination grounds, so the defendant's voluntary statements during the polygraph test cannot be barred on those grounds. (D) is incorrect because the Federal Rules do not specifically classify polygraph tests as irrelevant; in fact, polygraph evidence could be relevant under the Federal Rules. Relevant evidence is evidence having any tendency to make the existence of any fact that is of consequence to the determination of an action more probable than it would be without the evidence. [Fed. R. Evid. 401] Polygraph evidence is relevant under this test because it does tend to make the existence of a fact of consequence—whether the person is telling the truth—more probable. However, as discussed above, this relevant evidence can still be excluded under Federal Rule 403 if the court decides its probative value is substantially outweighed by the danger of unfair prejudice.

A defendant is charged with beating a victim to death with a set of brass knuckles during the course of a fight in a tavern. The victim was found to have a pistol on his person at the time of the fight. During the course of the trial, the defendant took the stand in his own defense and testified that the victim threatened him with a gun and the defendant had hit the victim with the brass knuckles in self-defense. To rebut the defendant's claim, the prosecution wishes to place the bartender on the stand, who will testify that two years prior to the attack on the victim, she had seen the defendant approach a customer in her tavern from behind, put on a pair of brass knuckles, and strike the customer a severe blow on the side of the face with a brass-knuckled fist. The prosecutor, in accordance with local court rules, has apprised the defense attorney of the general tenor of the bartender's proposed testimony. As soon as the bartender is sworn in, the defense attorney raises an objection. How should the court rule on the admissibility of the bartender's testimony? a. admissible, as substantive evidence that the defendant did not act in self-defense in beating the victim b. admissible, to attack the defendant's credibility c. inadmissible, because prior bad acts cannot be admitted to prove the defendant's propensity to commit the specific crime with which he is charged. d. inadmissible, because the defendant has not put his character in issue in this case

c. is correct. The bartender's testimony is inadmissible because the defendant's prior fight in the tavern cannot be admitted to prove his propensity to beat someone to death. The basic rule is that when a person is charged with one crime, extrinsic evidence of his other crimes or misconduct is inadmissible if such evidence is offered solely to establish a criminal disposition. [Fed. R. Evid. 404(b)] The danger is that the jury may convict because of past conduct rather than because of guilt of the offense charged. While evidence of other crimes is admissible if it is independently relevant to some other issue (e.g., motive, intent, or identity), the defendant's prior fight appears to have no relevance other than as evidence of his violent disposition. It is therefore inadmissible. (A) is incorrect because it suggests that the bartender's testimony should be admitted to show the defendant's propensity for violence. As stated above, extrinsic evidence of his prior misconduct is inadmissible if offered solely to establish a criminal disposition. Evidence of specific acts of the person in question as demonstrating that person's character is permitted only in the few instances when character is itself one of the ultimate issues in the case. [Fed. R. Evid. 405(b)] The defendant's propensity for violence is not an ultimate issue in this case. (B) is incorrect because extrinsic evidence of the defendant's previous bad acts cannot be used to impeach him. A specific act of misconduct must be probative of truthfulness (i.e., an act of deceit or lying) and can be elicited only on cross-examination of the witness. Extrinsic evidence is not permitted. Therefore, testimony concerning the defendant's prior incident is not admissible for impeachment. (D) is incorrect because it is irrelevant. It is true that the defendant has not put his character in issue in this case simply by pleading self-defense. Even if he had, however, the prosecutor could not rebut by having a witness testify as to prior instances of misconduct; only reputation or opinion evidence would be admissible.

A plaintiff sued a defendant for personal injuries, claiming that while the plaintiff was driving through an intersection at the posted speed limit, the defendant failed to stop at a stop sign and struck her car. At trial, the plaintiff calls the defendant's friend to testify to the fact that the defendant never stops at the stop sign at the accident intersection and invariably "runs" every stop sign. Should the defendant's objection to the testimony be sustained? a. yes, because it is not the best evidence. b. yes, because character evidence is inadmissible in a civil case. c. no, because it is evidence of habit d. no, because it is self-serving

c. is correct. The testimony is admissible as evidence of habit. Habit describes a person's regular response to a repeated specific situation. Evidence of a person's habit may be admitted to prove that on a particular occasion the person acted in accordance with that habit. [Fed. R. Evid. 406] According to the testimony, the defendant regularly fails to obey the stop sign at the intersection at which the collision occurred, and in fact, he regularly disregards any stop sign. This regular response to a specific circumstance constitutes a habit. Consequently, the testimony, which is evidence of this habit, is admissible, and so the defendant's objection should be overruled. (A) is incorrect because the best evidence rule is inapplicable to this question. Under the best evidence rule, where the terms of a writing are material, the original writing must be produced in proving the terms of the writing. Here, there is no writing material to the case; thus, the best evidence rule does not come into play. (B) is incorrect because the offered testimony is not character evidence. Character describes one's disposition with respect to traits or general patterns of behavior. If the testimony were that the defendant is generally a careless driver, it would be inadmissible character evidence. The testimony, however, describes a repeated response by the defendant to repeated specific circumstances, which is admissible habit evidence. (D) is incorrect because the fact that an objection is self-serving does not form a basis for overruling (or sustaining) the objection. In a sense, all objections are self-serving to the party making them, just as the evidence to which an objection is made is self-serving to the party offering it.

In its lead editorial in the Sunday edition, a suburban daily newspaper characterized a real estate developer as a "common thief." The developer promptly filed suit against the newspaper for defamation. During the course of the presentation of the plaintiff's case, he sought to put a witness on the stand who is prepared to testify that the plaintiff once saved the life of a fellow soldier in combat. If the newspaper's lawyer objects, should the court rule that the testimony is admissible? a. yes, because the plaintiff has a right to introduce evidence of his good character b. yes, because the plaintiff's character has been brought into question by the editorial c. no, because the witness's testimony is not probative of any material issue d. no, because specific instances of conduct are not admissible to prove character

c. is correct. The witness's testimony is inadmissible because it is not probative of a material issue (i.e., whether the plaintiff is a thief). Relevant evidence tends to prove or disprove a material fact in issue. Here, the testimony tends to prove that the plaintiff is brave and selfless, but it is not relevant as to the fact in issue, which is whether he is honest. (A) is incorrect because character evidence is admissible in a civil suit only when, as here, proof of a person's character, as a matter of substantive law, is an essential element of a claim or defense. However, even when character is in issue, the evidence must be relevant to the particular character trait in issue; here, it is not relevant to the issue of the plaintiff being a thief. (B) is incorrect for the same reason; to be admissible, the evidence must be relevant. (D) is incorrect because proof of specific instances of a person's conduct is admissible when character is directly in issue. [Fed. R. Evid. 405(b)]

Greg was in prison, serving a lifetime sentence for rape. While in prison, he got into a fight with another inmate. Both inmates pulled out homemade knives, and Greg was stabbed in the chest. He fell to the floor and was bleeding profusely. As the guards rushed in to help, Greg grabbed one of them and said: "Listen. I don't have much time left. You should know that I killed a person three years before I was convicted of rape. The person's name was Charles Grundy, and he lived in Albany, New York." Greg then fell unconscious. The prison staff tended to his wound, but he died two days later. The guard investigated the case of Charles Grundy, and found out that indeed he had been murdered a few years ago. He also learned that a person named Kyle Worthington had been convicted of the murder, though Worthington always claimed he was innocent and that someone else committed the murder. A few months later, Worthington's conviction was overturned on appeal, and he was re-tried. The guard was called to testify at Worthington's new trial to tell the jury that Greg had confessed to the crime. The prosecutor argued that the confession was hearsay. Is Greg's confession a dying declaration? a. Greg's confession is admissible as a dying declaration. b. Greg's confession is not admissible as a dying declaration because he did not die immediately after making the statement. c. Greg's confession is not admissible as a dying declaration because the content of the statement concerns a separate crime. d. Greg's confession is not admissible as a dying declaration because his statement does not have a guaranty of trustworthiness.

c. is correct. A dying declaration must be about the cause or circumstances of the declarant's imminent demise. Greg's statement is about someone else's death. a. is incorrect. Greg's confession is not a dying declaration because it does not meet all of the requirements of Rule 804(b)(2). b. is incorrect. For a statement to qualify as a dying declaration under Rule 804(b)(2), the declarant must believe that his or her death is imminent. It does not matter whether the declarant actually dies immediately or even whether he dies at all. d. is incorrect. There is no requirement that a dying declaration have a special guaranty of trustworthiness.

Samantha is suing her former employer, Dunaway Corporation, for age discrimination. Samantha alleges she was fired because Dunaway wanted their sales force to have a "younger look," and her manager Damon believed she was too old to work at the sales counter. Damon met with the Dunaway's Vice-President one week before she was fired, and Damon's secretary overheard the meeting while she was at her desk outside Damon's office. The secretary is willing to testify that she heard the Vice-President say: "Your saleswomen don't seem to match the image our company is trying to project," and Damon responded: "I know. I am going to let go some of the older saleswomen and I have some new college graduates I am interviewing for the open positions." Dunaway objects to this evidence. Damon is not a party to the lawsuit; Samantha is only suing Dunaway. Is Damon's statement admissible? a. Damon's statement is admissible only to impeach Damon if he testifies in a manner inconsistent with the statement. b. Damon's statement is admissible for the truth of the matter asserted as a business record under Rule 803(6). c. Damon's statement is admissible for the truth of the matter asserted as a party-opponent statement under Rule 801(d)(2). d. Damon's statement is admissible for the truth of the matter asserted as a statement against interest, but only if Damon is unavailable as a witness.

c. is correct. Although Damon himself is not a party to the lawsuit, he is an employee speaking within the scope of his employment, and therefore his statement is admissible as a party-opponent statement of Dunaway's under Rule 801(d)(2)(D). a. is incorrect. Although the statement is admissible to impeach, a hearsay exception applies. b. is incorrect. Under Rule 803(6), the statement must be a record made by someone in the organization, and the oral statement does not count as a "record." d. is incorrect. It is not clear that the statement was against Damon's interest at the time that he made it (it probably would have been against his interest had he disagreed with the Vice-President, but instead he agreed with him).

Ross was at work using a power saw that had been manufactured by Cutter Tools. Ross held the trigger of the saw steady with his left hand and used his right hand to guide the wood through the rotating blade. When the wood was cut, Ross leaned down to pick up the cut piece of wood and took his left hand off the trigger of the saw. However, the saw blade did not stop rotating, and as Ross leaned over to pick up the wood, his right hand came into contact with the saw blade and was severed from his arm. Ross sued Cutter Tools, alleging that their design was unreasonably dangerous because the saw blade did not immediately stop rotating when the user took his hand off the trigger. Which of the following is MOST likely to be admissible? a. Two weeks after the incident, Cutter Tools began manufacturing power saws with a locking mechanism so that the blade immediately stops rotating when the trigger is released. b. Ross had been cited at work three times over the last year for causing accidents resulting in injury or damage to property. c. Two weeks after the incident, the same model of the Cutter Tools power saw severed the finger of a woman using it in her home when the blade kept rotating after she took her finger off the trigger. d. One week before the trial, Cutter Tools offered to pay Ross $50,000 in exchange for dropping the lawsuit.

c. is correct. Because this was the same model saw, and because the accident happened in a very similar way, this evidence is relevant to proving the saw had a defect. It is irrelevant that the accident happened after the incident with Ross, because the evidence still tends to prove that the design itself was defective. a. is incorrect. This is a subsequent remedial measure, and is barred under Rule 407 if offered to prove a defect in the saw's design. It may be admissible if offered to impeach or to prove the feasibility of the new safety measure if Cutter Tools controverted the feasibility, but as far as we know there is no contradictory testimony to impeach and there is no way Cutter Tools could have contested feasibility. b. is incorrect. This is improper propensity evidence under Rule 404(a). The only relevance of his prior accidents would be to prove that Ross is a careless person and therefore was likely to be careless on this occasion, and that is an impermissible. d. is incorrect. This evidence would be barred by Rule 408, since it is evidence of a settlement offer and there was a disputed claim at the time the offer was made.

Tamara was arrested for domestic violence assault after she allegedly hit her husband in the face with a lamp. Before she was arrested, she told the police officers: "Sure I hit him. He had been yelling at me for twenty minutes, and I was afraid he was going to become violent, so I took matters into my own hands." At trial, the prosecutor seeks to admit her statement to the jury in order to prove that she hit her husband. Tamara's attorney objects that the statement is hearsay. Assume no Miranda issues apply. Is Tamara's statement admissible? a. No, it is barred by the hearsay rule because it was not against her interest at the time she said it. b. No, it is barred by the hearsay rule unless Tamara chooses to take the stand to explain the statement. c. Yes, it is exempt from the hearsay rule as a party-opponent statement. d. Yes, the hearsay rule does not apply because the statement is not being offered for the truth of the matter asserted.

c. is correct. Every statement made by an opposing party is exempt from the hearsay rule, whether or not she testifies and whether or not it was against her interest when she said it. a. is incorrect. Rule 804(b)(3), which admits statements that are against the declarant's interest at the time they are made, does not apply in this context. b. is incorrect. Although there are some hearsay exceptions which only apply if the declarant takes the stand (such as Rule 801(d)(1) or 803(5)), this statement does not fall into any of those exceptions. d. is incorrect. The prosecutor is offering this statement to prove that Tamara hit her husband—that is, to prove the truth of the matter asserted.

Louis was pulled over for speeding by Officer Gareth of the Tarrytown Police Department. During this encounter, Officer Gareth allegedly ordered Louis out of the car and then struck him six times with his nightstick when Louis insulted him. Louis is now suing Officer Gareth and the Tarrytown Police Department, alleging that Officer Gareth used excessive force against him and that the Tarrytown police department failed to properly train, supervise, and discipline Officer Gareth. In his case-in-chief, Louis seeks to admit evidence that on three prior occasions in the past year, Officer Gareth had beaten motorists after he pulled them over for speeding. This evidence is: a. Admissible against Officer Gareth to prove he has a propensity for violence and against the Tarrytown Police Department. b. Admissible against Officer Gareth to prove he has a propensity for violence, but not against the Tarrytown Police Department. c. Admissible against the Tarrytown Police Department, but not admissible against Officer Gareth if offered to prove he has a propensity for violence. d. Inadmissible against both Officer Gareth if offered to prove he has a propensity for violence and inadmissible against the Tarrytown Police Department.

c. is correct. If the evidence is offered to prove that Officer Gareth has acted violently in the past and is therefore more likely to have acted violently on this occasion, it is improper character evidence and is barred by Rule 404. However, it is relevant in the case against the police department as evidence that the department knew (or should have known) that Officer Gareth was violent and therefore should have taken some measures (training, discipline, termination) to prevent him from acting violently again. In other words, the police department could be liable for negligent entrustment by not taking action against Office Gareth. Thus, Officer Gareth's character is directly at issue in the case against the police department, and the evidence is not barred by Rule 404. a. is incorrect. If the evidence is offered to prove that Officer Gareth has acted violently in the past and is therefore more likely to have acted violently on this occasion, it is improper character evidence and is barred by Rule 404. It is not specific enough nor repeated enough to be considered habit evidence under Rule 406. b. is incorrect. If the evidence is offered to prove that Officer Gareth has acted violently in the past and is therefore more likely to have acted violently on this occasion, it is improper character evidence and is barred by Rule 404. It is not specific enough nor repeated enough to be considered habit evidence under Rule 406. d. is incorrect. The evidence in the case against the police department is relevant to show the department knew (or should have known) that Officer Gareth was violent and therefore should have taken some measures (training, discipline, termination) to prevent him from acting violently again. Thus, Officer Gareth's character is directly at issue in the case against the police department, and the evidence is not barred by Rule 404.

After reviewing Samantha's tax returns for the past three years, the Internal Revenue Service ("IRS") contacted Samantha and asked her to come to the local IRS office to explain some irregularities. The IRS agent suspected that Samantha had intentionally understated her income by over $100,000 and was hoping that he could elicit some incriminating statements from her during the meeting. At the meeting, the IRS agent presented Samantha with the evidence of the discrepancies between her actual income and the income she reported on her taxes, and he told her that she faced significant civil penalties. Samantha admitted that she made a mistake in calculating her taxes, but claimed it was merely an oversight. She offered to pay the minimum fine in order to avoid additional penalties. The IRS agent refused the offer, and the meeting ended. The IRS turned the case file over to the United States Attorney's office, and Samantha was ultimately criminally charged with income tax evasion. At trial, the government sought to admit her statements in which she confessed to understating her income and offered to pay the minimum fine. Samantha objects. Are Samantha's statements admissible? a. The statements and the offer are inadmissible under Rule 408 because Samantha was conducting settlement negotiations. b. The statements and the offer are inadmissible under Rule 410 because Samantha was conducting plea negotiations. c. Samantha's offer to pay the maximum civil penalty is inadmissible, but her statement admitting to understating her income is admissible. d. All of the statements are admissible.

c. is correct. Rule 408 applies in this case, because there was a disputed claim and Samantha was conducting settlement negotiations. However, if a defendant made statements when negotiating a claim by a public office exercising its enforcement authority, those statements are admissible in a subsequent criminal trial under Rule 408(a)(2). The offer to settle itself is still inadmissible under Rule 408(a)(1). a. is incorrect. Although there was a disputed claim, and Samantha was conducting settlement negotiations, Rule 408(a)(2) does not bar the use of statements that are made when negotiating a claim by a public agency when offered in a subsequent criminal trial. b. is incorrect. At the time Samantha made the statement, there were no criminal charges pending, and Samantha never offered to plead guilty, so this was not a plea negotiation and Rule 410 does not apply. d. is incorrect. Samantha's offer to pay occurred during a settlement negotiation, and so Rule 408 applies to preclude the offer.

Which of the following would most likely be INADMISSIBLE? a. In order to prove consciousness of innocence, a criminal defendant offers evidence that he rejected a prosecutor's offer during plea bargaining. b. The plaintiff was injured by the defendant's employees when they were working on a construction site and the plaintiff sues the defendant. In his case-in-chief, the defendant claims that he saw no need to upgrade the safety protocols for his employees. In rebuttal, the plaintiff offers evidence that the defendant actually did upgrade his safety protocols after the incident. The plaintiff offers this evidence to impeach the defendant's testimony. c. Plaintiff is suing defendant for injuries sustained when the defendant hit her with his car. During settlement negotiations, the defendant admits that he may have been texting at the time of the accident, and he is willing to pay her half of what she is suing for to settle the case. Plaintiff refuses. During the defendant's case-in-chief, he testifies that he was not texting at the time of the accident. The plaintiff seeks to admit the statement defendant made during the settlement negotiations in order to impeach the defendant's testimony. d. In a criminal case, a defendant is brought back to the precinct and the police read him his Miranda rights, fulfilling the requirements of the Fifth Amendment. After he waived those rights, he gave a full confession to the crime. At trial, the prosecutor seeks to admit the confession to prove that the defendant committed the crime.

c. is correct. Rule 408 bars all statements made during compromise negotiations if offered to prove or disprove the validity or amount of a disputed claim or to impeach by a prior inconsistent statement or contradiction. In this case the plaintiff is seeking to admit a protected statement in order to impeach the defendant with a prior inconsistent statement. a. is incorrect. Rule 410 bars evidence of plea offers, but it only bars the prosecutor from offering such evidence. It does not prevent the defendant from admitting evidence of a plea offer. This evidence is relevant because it has some probative value to prove consciousness of innocence. b. is incorrect. Rule 407 bars evidence of subsequent remedial measures if offered to prove fault or liability, but allows evidence of subsequent remedial measures if offered to impeach. d. is incorrect. There is no rule of evidence that bars this confession. It is not a statement made during plea negotiations under Rule 410, and it is not hearsay because it is a party-opponent statement under Rule 802(d)(2).

Sandy is arrested by the police for robbery. In which of the following situation is Sandy's confession LEAST likely to be admissible if offered by the prosecutor against Sandy? (Assume all Miranda rules have been complied with; i.e., there are no constitutional problems with any of the statements). a. A few hours after the arrest, a police officer tells Sandy that she should "come clean," and if she tells the truth now, the prosecutor "might cut you a deal." Sandy confesses to the robbery. b. A few hours after the arrest, Sandy meets with the prosecutor. The prosecutor says: "You're in a lot of trouble. I may be able to help you out, but I need to know your side of the story." Sandy confesses to the robbery. c. A few hours after the arrest, a police officer tells Sandy: "I just got off the phone with the prosecutor. She told me that she would be willing to charge you with a misdemeanor if you plead guilty at the arraignment and tell us right now the name of the person who committed this robbery with you." The police officer is lying—he has not spoken to the prosecutor—but there is no way for Sandy to know this. Sandy confesses to the robbery. d. A few hours after the arrest, Sandy meets with the prosecutor. The prosecutor says: "We have your partner in the next room, and he is telling us everything about what happened. He's hanging you out to dry. If I were you, I would start cooperating too—otherwise you are going to be left out in the cold." The prosecutor is lying—the partner has not been apprehended—but there is no way for Sandy to know this. Sandy confesses to the robbery.

c. is correct. Rule 410 will probably bar these statements. As long as the defendant reasonably believes she is engaging in a plea bargain, Rule 410 will protect his statements. In this case, although the police officer does not have the authority to make a bargain, the defendant does not know this, and his belief that he is plea bargaining is reasonable. a. is incorrect. Rule 410 does not bar these statements. Although the police officer makes vague promises of a deal if Sandy confesses, there is no consideration offered by the officer and the defendant never offers to plead guilty. Also, the police officer has no authority to plea bargain with Sandy. b. is incorrect. Rule 410 probably does not bar these statements. Although the prosecutor makes a vague promise to "help out," there is no concrete consideration offered by the prosecutor (such as a reduction in the charge or in the recommended sentence) and the defendant never offers to plead guilty. d. is incorrect. Rule 410 probably does not bar these statements. The prosecutor never makes any specific offer to the defendant; she only states that the defendant should "start cooperating." There is no concrete consideration offered by the prosecutor (such as a reduction in the charge or in the recommended sentence) and the defendant never offers to plead guilty.

The plaintiff in a civil case called Oliver to testify. On cross-examination, the defendant's attorney asked Oliver if it was true that on September 15th of the previous year, he filed a fraudulent claim for insurance. Oliver denied that he did so. The defendant's attorney then admitted an affidavit signed by Oliver in which he admitted to the insurance company that his September 15th claim was fraudulent. The defendant's attorney admitted the affidavit to prove not only that Oliver filed the fraudulent claim (and thus had a propensity for dishonesty) but also to prove that Oliver had just lied to the jury when he denied filing it. Assume the plaintiff objected to all of the defendant's questions and actions during this time. At what point (if any) should the judge have sustained the plaintiff's objections and cut off this line of inquiry? a. None; the defendant's questions and actions conformed to the rules of evidence. b. The defendant should not have been allowed to ask about the fraudulent claim at all, because it is evidence of a specific action, not reputation or opinion evidence. c. The defendant should not have been allowed to admit the affidavit, since it is extrinsic evidence. d. The judge should have allowed the defendant to ask all the questions and admit the affidavit, but should then have given a limiting instruction directing the jury to consider the affidavit only as evidence that Oliver lied to the jury at trial about his past action; not as evidence that he committed fraud on September 15th.

c. is correct. Rule 608(b) only allows an opposing party to ask questions about specific acts on cross-examination. The opposing party is not allowed to admit extrinsic evidence of those specific acts. a. is incorrect. At least some of the defendant's actions were impermissible under Rule 608(b). b. is incorrect. Under Rule 608(b), an opposing party is permitted to ask questions about specific acts on cross-examination as long as the specific acts are relevant to credibility. d. is incorrect. The affidavit is inadmissible because it is extrinsic evidence, and Rule 608(b) does not permit extrinsic evidence of specific dishonest acts.

Sam is charged with breaking into a safe in his supervisor's office in June and stealing merchandise that his supervisor stored there. Only two people—the supervisor and the private security officer who installed the safe—supposedly knew the combination to the safe. At trial, the supervisor wishes to testify that he believes Sam broke into the safe in April, two months before the incident in question. He thinks this is true because he came to work early one day and saw Sam in his office for no reason, and then he checked his safe later that day and items had been moved around. Because nothing was stolen, he never reported this incident to the police. The prosecutor argues that Sam's alleged entry into the safe in April is admissible under Rule 404(b), because it shows that Sam had the knowledge required to break into the safe (either he somehow had the combination or was able to crack the safe); and the fact that he had this knowledge makes it more likely that he broke into the safe in June. Sam's attorney concedes that if Sam had broken into the safe in April, the supervisor should be able to testify to that fact, but he argues that there is insufficient evidence that Sam did in fact break into the safe. Should the court admit the evidence that Sam allegedly broke into the safe in April? a. Preclude the evidence because Sam was not convicted of the prior crime. b. Admit the evidence if the judge is convinced by a preponderance of the evidence that Sam committed the prior crime. c. Admit the evidence if the judge believes a reasonable jury could find by a preponderance of the evidence that Sam committed the prior crime. d. Admit the evidence if the judge is convinced beyond a reasonable doubt that Sam committed the prior crime.

c. is correct. Sam's alleged prior crime is a question of conditional relevance, and so is governed by Rule 104(b). Although Rule 104(a) states that generally the question of admissibility depends on the judge's own decision, Rule 104(b) states that there need only be proof sufficient to support a finding that the fact does exist. The Huddleston case states that this means that there needs to be proof sufficient to support a finding by the jury that the fact does exist. a. is incorrect. Sam's alleged prior crime is a question of conditional relevance, and so is governed by Rule 104(b). Rule 104(b) has no requirement that the fact being offered has been found to be true in a prior case. b. is incorrect. Sam's alleged prior crime is a question of conditional relevance, and so is governed by Rule 104(b). Although Rule 104(a) states that generally the question of admissibility is determined by the judge, Rule 104(b) states that when the relevance of evidence is conditioned on proof of a fact, there need only be proof sufficient to support a finding that the fact does exist. d. is incorrect. Sam's alleged prior crime is a question of conditional relevance, and so is governed by Rule 104(b). Rule 104(b) has no requirement that the fact being offered has been proven beyond a reasonable doubt.

Granger Machines manufactures farm equipment and vehicles. John Kerling, a farmer, owned a harvester that had been manufactured by Granger. On January 5th, the harvester malfunctioned while Kerling was using it, and Kerling's arm was cut off. One week later, on January 12th, Granger sent out a recall letter telling all owners of the harvester to take them into the local Granger dealer, to add on a safety shunt that would prevent such accidents in the future. Daniel Sampson was another farmer who owned a Granger harvester. On January 13th, Sampson's hand was cut off by the harvester. Although the recall letter had been sent out at that point, Sampson had not yet received it. On January 15th, Kerling and Sampson each separately sued Granger Machines, alleging that the harvester had a design defect. In their respective trials, Kerling and Sampson each attempted to admit the recall letter sent out by Granger as evidence that the harvester was defective. Is the recall letter admissible for this purpose? a. Yes, it is admissible in both Kerling's case against Granger and in Sampson's case against Granger. b. It is admissible in Kerling's case against Granger, but not Sampson's case against Granger. c. It is admissible in Sampson's case against Granger, but not Kerling's case against Granger. d. No, it is inadmissible in both Kerling's case against Granger and in Sampson's case against Granger.

c. is correct. The letter is a subsequent remedial measure with regard to Kerling, because it was an action taken by the defendant to make the harvester safer, and it was issued subsequent to the injury to Kerling. Thus, under Rule 407 it is inadmissible in Kerling's case if offered to prove the existence of a design defect. However, the letter was issued before the incident which led to Sampson's injury, so it is not a subsequent remedial measure with regard to Sampson and therefore will be admissible. a. is incorrect. The letter is a subsequent remedial measure with regard to Kerling, because it was an action taken by the defendant to make the harvester safer, and it was issued subsequent to the injury to Kerling. Thus, under Rule 407 it is inadmissible in Kerling's case if offered to prove the existence of a design defect. b. is incorrect. The letter is a subsequent remedial measure with regard to Kerling, because it was an action taken by the defendant to make the harvester safer, and it was issued subsequent to the injury to Kerling. Thus, under Rule 407 it is inadmissible in Kerling's case if offered to prove the existence of a design defect. d. is incorrect. Although Rule 407 bars evidence of a subsequent remedial measure if offered to prove culpable conduct, the letter was issued before the incident which led to Sampson's injury. Thus, it is not a subsequent remedial measure with regard to Sampson and therefore will be admissible when Sampson sues Granger.

Michael is on trial for murder. The prosecutor's theory of the case is that Michael stabbed his roommate six times in the chest while his roommate was sleeping. The prosecutor wishes to admit three photographs of the crime scene as it appeared when officers first arrived, showing the victim in the bed, with the stab wounds clearly visible and blood soaking through his sheets and nightclothes. Michael's attorney objects to the photos, arguing that they will unfairly prejudice the jury. Should the judge preclude the photos? a. The judge should preclude the evidence if she determines that it is likely to cause an emotional reaction in the jury. b. The judge should preclude the evidence if she determines that it carries any risk of unfair prejudice. c. The judge should preclude the evidence if the risk of unfair prejudice substantially outweighs the probative value of the evidence. d. The judge should preclude the evidence if there is any other way to prove the fact of death that is less prejudicial.

c. is correct. This is the correct test under Rule 403. a. is incorrect. Although an emotional reaction could mean that there is high level of unfair prejudice, the mere fact that a piece of evidence may cause an emotional reaction is not a reason to preclude it. b. is incorrect. Rule 403 states that a piece of evidence will be precluded only if the unfair prejudice substantially outweighs its probative value. The mere fact that the evidence may have some risk of unfair prejudice is not sufficient to require preclusion. d. is incorrect. Rule 403 does not require a party to use the least unfairly prejudicial evidence; it only states that the level of unfair prejudice created by the evidence should not substantially outweigh its probative value.

Which of the following pieces of evidence is most likely to be admissible to impeach the witness? a. Stephanie is on trial for assault. She testifies on her own behalf. The prosecutor seeks to admit her prior conviction for aggravated assault, a felony, from five years ago. b. George is on trial for attempted murder. He testifies on his own behalf. The prosecutor seeks to admit his prior conviction for selling cocaine, a felony, from three years ago. c. Fred is on trial for burglary. He does not testify, but Victoria testifies for the defense, telling the jury that Fred was with her all night at the night of the burglary and so could not have committed the crime. The prosecutor seeks to admit Victoria's prior conviction for selling cocaine, a felony, from five years ago. d. Harriett is on trial for grand larceny. She does not testify, but Henry testifies for the defense, telling the jury that he saw someone else commit the crime. The prosecutor seeks to admit Henry's prior conviction for vandalism, a misdemeanor, from two years ago.

c. is correct. Under Rule 609(a)(1)(A), the appropriate standard for admitting a prior non-falsity felony conviction from within ten years for any witness other than a criminal defendant is the Rule 403 standard. This is the easiest standard from all of the options available. a. is incorrect. Since the criminal defendant is the witness, Rule 609(a)(1)(B) applies, which means that the evidence should only be admitted if the probative value of the evidence outweighs its prejudicial effect. This is a harder standard to meet than if a witness other than the accused is being impeached. Also, the prior conviction is identical to the crime for which he is now on trial, so the unfair prejudice is higher for this fact pattern than for any of the other options. b. is incorrect. Since the criminal defendant is the witness, Rule 609(a)(1)(B) applies, which means that the evidence should only be admitted if the probative value of the evidence outweighs its prejudicial effect. This is a harder standard to meet than if a witness other than the accused is being impeached. Also, selling cocaine has a very low probative value for impeachment purposes. d. is incorrect. Under Rule 609, a non-falsity misdemeanor conviction is not admissible for impeachment purposes.

A plaintiff brought a personal injury action against a defendant, the owner of a small fishing resort, for injuries he suffered when a dockside chair he was sitting on collapsed. At trial, the plaintiff testified that he had reported to the defendant the previous day that one of the chairs had a loose leg, whereupon the defendant tightened the screws holding the leg to the chair body, but that the next day the repaired leg of the chair collapsed while the plaintiff was fishing from it, injuring him. The plaintiff now wishes to offer evidence showing that the defendant had attached a new chair leg after the accident. Should the defendant's objection to that evidence be sustained? a. no, because it tends to prove the defendant's negligence b. no, because it is relevant to the defendant's state of mind c. yes, because it constitutes assertive conduct. d. yes, for public policy reasons

d. is correct Evidence that the defendant had attached a new chair leg after the accident is inadmissible because, for public policy reasons, evidence of repairs or other precautionary measures made after an injury is inadmissible to prove negligence or culpable conduct. [Fed. R. Evid. 407] The purpose of this rule is to encourage people to make such repairs. Here, the plaintiff is offering the evidence to prove the defendant's negligence in the original repair of the chair, by showing the need to attach a new leg. Thus, this evidence is inadmissible, and the objection should be sustained. (A) is wrong because, as discussed above, the evidence may not be used to show the defendant's negligence. Thus, the tendency of the evidence to prove negligence would constitute a reason for sustaining the objection, rather than overruling it. (B) is wrong because the defendant's state of mind is not at issue. In addition, the proffered evidence does not really tend to prove anything relative to her state of mind. (C) is wrong for two reasons: (i) Even if the act of replacing the chair leg constituted assertive conduct, it would not be hearsay. If the defendant's conduct was a statement, it would be a statement of a party-opponent, and thus it would be nonhearsay. (ii) The act of replacing the leg is not assertive conduct constituting a statement under the hearsay rule. Assertive conduct is conduct intended by the actor to be a substitute for words. The defendant was not trying to communicate anything by replacing the leg.

Dana is on trial for child abuse. The prosecution's theory is that on October 5th, Dana got angry at her six-year-old son for having stained the carpet in her bedroom, and so she struck him multiple times, causing significant bruising to his face and body. Which of the following evidence (if any) should be excluded as IRRELEVANT to the prosecutor's case? a. One year ago, Dana's son was removed from her by the state for six months because Dana hit him in the face with a coffee mug and broke his jaw. b. Dana did not send her son to school for five days after October 5th. c. On October 12th, Dana's son told his teacher at school that a week before his mother had hit him and made him cry. d. All of the above facts are relevant to the case.

d. is correct. All of the above facts are relevant because they have a tendency to make a material fact (that Dana did indeed strike her son) more probable than it would be without the evidence. Some of this evidence is barred as propensity evidence or by the hearsay rule, but the question asks about relevance, not admissibility. a. is incorrect. The fact that Dana used excessive force against her child on a prior occasion makes it more likely she would use excessive force against the child on the occasion in question. If this were offered as propensity evidence, it would be inadmissible Rule 404, but the question asks about relevance, not admissibility. This prior conduct is relevant, as it tends to prove a fact in consequence. b. is incorrect. Although there are many possible reasons why Dana may not have sent her child to school for five days, one possible reason is that she kept him home so that nobody could see his bruises. Thus, this piece of evidence has a tendency to make a material fact more probable than it would be without the evidence. c. is incorrect. The fact that Dana's son told someone that Dana committed this act is relevant. This evidence will probably be precluded as hearsay, since there is no hearsay exception in the Federal Rules for this statement, but the question asks about relevance, not admissibility.

Stan is charged with felony domestic violence after he allegedly struck his girlfriend in the chest with a baseball bat. At trial, the prosecutor will seek to offer evidence that six months earlier, Stan had beaten his former girlfriend with his fists so severely that she went to the hospital for two days. Stan is pleading self-defense, and at trial he will seek to admit evidence that three years earlier, his girlfriend got into a dispute with a store manager while she was shopping and she pushed the manager into a glass table, causing multiple lacerations and requiring the manager to get over twenty stitches. There is no evidence that Stan knew about the incident with the store manager at the time he struck his girlfriend with the baseball bat. How should the judge rule on these proposed pieces of evidence? a. Both pieces of evidence are admissible regardless of the order in which they are offered. b. The evidence that Stan's girlfriend pushed the manager into a glass table three years ago is admissible, but the evidence that Stan beat his former girlfriend six months earlier is not admissible. c. The evidence that Stan's girlfriend pushed the manager into a glass table three years ago is admissible, and after that evidence is admitted, the evidence that Stan beat his former girlfriend six months earlier is admissible. d. Both pieces of evidence are inadmissible regardless of the order in which they are offered.

d. is correct. Although Rule 404(a)(2) allows the criminal defendant to go first in offering propensity evidence of the victim, and then allows the prosecutor to respond with evidence of the defendant's propensity, Rule 405 states that when character evidence is offered to prove propensity (as here where Stan is proving that his girlfriend has a propensity to be violent, and therefore is more likely to have been acting violently on this occasion), character evidence can only be proven on direct with reputation or opinion evidence, not with specific instances of conduct. Stan could try to argue that his girlfriend's violent action towards the store manager is admissible to support his self-defense claim, to show he had reasonable fear of her when he hit her with the bat, but since he had no knowledge of his girlfriend's violent action at the time he struck her, this evidence is irrelevant to his state of mind at the time of the alleged crime. a. is incorrect. Rule 404(a)(2) and Rule 405 set out specific rules for admitting character evidence, stating that the criminal defendant must go first with offering character evidence and restricting the manner in which character evidence can be proved to the jury. b. is incorrect. Rule 405 states that when character evidence is offered to prove propensity (as here where Stan is proving that his girlfriend has a propensity to be violent, and therefore is more likely to have been acting violently on this occasion), character evidence can only be proven on direct with reputation or opinion evidence, not with specific instances of conduct. Stan could try to argue that his girlfriend's violent action towards the store manager is admissible to support his self-defense claim, to show he had reasonable fear of her when he hit her with the bat, but since he had no knowledge of his girlfriend's violent action at the time he struck her, this evidence is irrelevant to his state of mind at the time of the alleged crime. c. is incorrect. Rule 405 states that when character evidence is offered to prove propensity (as here where Stan is proving that his girlfriend has a propensity to be violent, and therefore is more likely to have been acting violently on this occasion), character evidence can only be proven on direct with reputation or opinion evidence, not with specific instances of conduct. Stan could try to argue that his girlfriend's violent action towards the store manager is admissible to support his self-defense claim, to show he had reasonable fear of her when he hit her with the bat, but since he had no knowledge of his girlfriend's violent action at the time he struck her, this evidence is irrelevant to his state of mind at the time of the alleged crime.

Westin Coal owns and operates dozens of coal mines in the state. One day there was a cave-in on one of their mines, and three miners died. The miners' families are now suing Westin, arguing that the mines were negligently maintained. In their case-in-chief, the plaintiffs called a former Westin employee, who testified that no safety inspection had been conducted on the mine for over two years. During the defendant's case-in-chief, Westin called an employee from Prescott Insurance, a large national insurance company. The Prescott employee will testify that Westin has an insurance policy with Prescott, and that as part of that policy, Prescott employees conduct independent safety inspections on all of the Westin mines every three months. The Prescott employee will then authenticate and admit a business record from Prescott's files which confirms that Prescott conducted regular safety inspections up until the time of the cave-in. The plaintiffs object to the testimony of the Prescott employee. This evidence is: a. Inadmissible as evidence of insurance under Rule 411. b. Inadmissible as a subsequent remedial measure under Rule 407. c. Inadmissible under Rule 402 because it is irrelevant. d. Admissible

d. is correct. Although Rule 411 prohibits evidence that a party was or was not insured, it only prohibits the evidence if it is used to prove that the party acted negligently or otherwise wrongfully. Here, Westin is offering the evidence of insurance as proof that the safety inspections were in fact conducted. a. is incorrect. Although Rule 411 prohibits evidence that a party was or was not insured, it only prohibits the evidence if it is used to prove that the party acted negligently or otherwise wrongfully. Here, Westin is offering the evidence of insurance as proof that safety inspections were indeed conducted. b. is incorrect. Rule 407 only prohibits remedial measures that occur subsequent to the incident; here, the defendant is offering evidence that the safety inspections occurred before the cave-in occurred. c. is incorrect. Rule 402 states that all irrelevant evidence is inadmissible. Here, the fact that regular safety inspections were conducted makes it less likely that the defendant was negligent in maintaining the mine, and so it is relevant.

Damon is on trial for vehicular manslaughter after he allegedly killed a ten-year-old child while driving. The prosecutor has three witnesses. The father of the victim will testify that he saw Damon swerve off the road and up onto the lawn, where he struck the child. The police officer who responded to the scene will testify that Damon was crying and distraught a few minutes after the incident, and said that he only looked away from the road for a second to check his cell phone. A coroner will also testify that he examined the child's body, and that the cause of death was massive trauma to the chest, likely caused by a car. Damon objects to the father's testimony as unfairly prejudicial. He notes that the father will no doubt be very emotional when he testifies about his son's death, and that this extreme amount of emotion will unfairly influence the jury. Damon is willing to agree to admit that he swerved off the road, up onto the lawn, and struck the child. Damon says that once he has admitted this fact, the father's testimony has little extra probative value and should be precluded. The prosecutor still wants the father to testify. What should the court do? a. Preclude the father from testifying even if Damon does not admit to these facts, because the unfair prejudice of his testimony in any form substantially outweighs its probative value. b. Once Damon has admitted to these facts, the father should be precluded from testifying. c. Allow the father to testify, but give the jury a limiting instruction that the father's testimony may not be reliable because of his close connection to the victim. d. Allow the father to testify with no limiting instruction.

d. is correct. Although there will be some level of unfair prejudice when the jury sees the father react emotionally on the stand, his testimony is extremely probative because he is an eyewitness to the event. Courts routinely allow victims and others with close relationships to the case testify even though they may display emotion; the unfair prejudice does not substantially outweigh the probative value of this testimony. Furthermore, in Old Chief v. United States (U.S. 1997), the Supreme Court stated that a party should only forced to accept an admission instead of offering live testimony if the probative value of the live testimony is extremely low. a. is incorrect. The father's testimony is extremely probative, since he is an eyewitness to the incident, and although his testimony will carry some danger of unfair prejudice, the danger does not substantially outweigh the probative value. b. is incorrect. In Old Chief v. United States (U.S. 1997), the Supreme Court stated that a party should only forced to accept an admission instead of offering live testimony if the probative value of the live testimony is extremely low. In this case, the probative value of the father's testimony is extremely high, since he is an eyewitness to the incident, and the prosecutor has the right to present his case with persuasive evidence. c. is incorrect. There is no reason to believe that the father's testimony will be unreliable because of his relationship to the victim, only that it will be emotional. And judges are precluded from commenting on the credibility of witnesses under Rule 605. Any bias that the father might have can be explored on cross-examination.

The First Union Bank was robbed on September 1. The robbers fled in a car that was seen driving into an area of the city known as the Downs. Officers found the car, abandoned, two hours later in the Downs. The detective in charge of the case reviewed the footage from the bank's security cameras and found one second of film in which one of the perpetrator's faces was uncovered and visible. The detective made a still photo from this one second of footage and then gave it to dozens of officers, who went door to door through the Downs, telling people that they were investigating a crime and asking them if they recognized the photo. Officer Brown was carrying out this assignment when he knocked on the door of Beverly Grogan. He showed her the photo and she gasped, putting her hand to her mouth and saying: "My God, that's my husband!" She then started crying. Officer Brown reported this to the detective, and her husband was ultimately arrested for robbing the bank. At trial, the prosecutor called Officer Brown to the stand and asked him what Beverly Grogan said when he showed her the photo of the bank robber in order to prove that the defendant was the person who robbed the bank. The defendant objects. What is the proper ruling on defendant's objection? a. Sustain the objection, because Beverly's statement is hearsay. b. Overrule the objection, because the statement is not being used to prove the truth of the matter asserted. c. Overrule the objection, because the statement is about a then-existing mental, emotional, or physical condition. d. Overrule the objection, because the statement is an excited utterance.

d. is correct. At the time the witness made the statement, the declarant was under the emotional stress of seeing her husband in a photo being held by a police officer—she seemed surprised, she exclaimed, and she began crying. Thus, it is an excited utterance under Rule 803(2). a. is incorrect. Although the statement is hearsay (because it is an out-of-court statement being offered to prove the truth of the matter asserted -that her husband is in fact the person in the photo), it is admissible under a hearsay exception. b. is incorrect. The statement is hearsay, because it is an out-of-court statement being offered to prove the truth of the matter asserted -that her husband is in fact the person in the photo. c. is incorrect. The statement does not concern the declarant's feelings, thoughts, or beliefs, so it is not a statement about a then-existing mental, emotional, or physical condition under 803(3).

Harrison Oil runs ten large oil drilling rigs in the Gulf of Mexico. On July 12, a hurricane hit the region, and the high winds damaged one of the rigs, causing an explosion which killed two workers. The families of the workers are now suing Harrison Oil for wrongful death based on alleged negligent operation of the rigs. Harrison Oil kept the rigs running through the hurricane, even though the winds reached 85 miles per hour. One week after the hurricane, Harrison Oil changed its safety protocols so that it now shuts down its oil rigs anytime the winds get above hurricane force, around 74 miles per hour. During the plaintiffs' case-in-chief, the plaintiffs called an engineering expert who testified that the accident would not have occurred if Harrison Oil had shut their rig down before the hurricane and stored the more fragile equipment indoors. Another expert testified that the industry standard is to shut down oil rigs whenever winds reach hurricane force. During the defendant's case, Harrison Oil's Vice-President of Operations testified on direct that the company followed all necessary safety procedures during the hurricane and that the explosion was an unavoidable accident. He also testified that although it was common in the industry to shut down oil rigs during hurricanes, it would not make sense for Harrison to do so because it uses older model oil rigs which take six hours and dozens of employees to shut down the rig, which would endanger the employees' lives even more than if the rig were left running. The plaintiffs now wish to introduce evidence that Harrison Oil changed its safety protocols so that it now shuts down its oil rigs during hurricane force winds. This evidence will be: a. Inadmissible. b. Admissible after the defendant's Vice-President has testified to prove that Harrison Oil was negligent for not following this procedure on July 12th when the workers were killed. c. Admissible only after the plaintiffs' experts testify, and only to prove that it would be feasible for Harrison Oil to follow this procedure. d. Admissible only after the defendant's Vice-President has testified, and only to prove that it would be feasible for Harrison Oil to follow this procedure.

d. is correct. Rule 407 allows evidence of subsequent remedial measures to prove feasibility if feasibility is controverted, and the defendant's testimony has controverted feasibility. a. is incorrect. Although the defendant's change in their safety protocols is a subsequent remedial measure, Rule 407 only bars subsequent remedial measures when offered to prove liability. Here, the defendant's Vice-President contested feasibility with his testimony, and so evidence of the subsequent remedial measure is admissible to prove feasibility. b. is incorrect. The subsequent remedial measure would probably be admissible to prove feasibility after the Vice-President contested feasibility, but Rule 407 bars evidence of subsequent remedial measures when offered to prove negligent conduct. c. is incorrect. Although Rule 407 allows evidence of subsequent remedial measures to prove feasibility, plaintiffs can only prove feasibility in this way once feasibility has been controverted by the opposing party. The defendant has not yet had a chance to contest feasibility.

Jarvis was driving home late at night when he struck Charles, who was on a bicycle. Charles was severely injured. The police were called and they gave Jarvis a Breathalyzer test. After he blew a .15—well over the legal limit—he was arrested and charged with driving while intoxicated. The next week Charles sued Jarvis, alleging that Jarvis was negligent, and seeking $100,000 in damages. One week before the criminal trial was set to begin, Jarvis offered to pay Charles $75,000 to settle the civil case. Will the prosecutor be able to admit the evidence of this offer in the criminal case against Jarvis? a. Yes, but only if Charles accepted the offer and settled the case. b. Yes, but only if Charles refused the offer and did not settle the case. c. Yes, because the rule barring settlement offers only applies to civil cases, not criminal cases. d. Yes, if Jarvis told Charles that part of the conditions of the settlement would be that Charles refused to cooperate with the prosecutor in the criminal case.

d. is correct. Rule 408 bars evidence of any settlement offer if offered to prove liability, but it specifically states that 408 does not apply if the evidence is offered to prove an effort to obstruct a criminal investigation or prosecution. Telling a witness in a criminal case that he cannot get a settlement unless he refuses to testify in a criminal case is an effort to obstruct the prosecution. This evidence is relevant to the criminal case against Jarvis because it tends to show he tried to obstruct the investigation because he believed himself to be guilty. a. is incorrect. Rule 408 bars both rejected and accepted offers. b. is incorrect. Rule 408 bars both rejected and accepted offers. c. is incorrect. Rule 408 bars any evidence of settlement negotiations in any future criminal case (unless the statement was made to a public agency and is then offered in a criminal case, which is not true here).

Which of the following out-of-court statements is most likely to be admissible in its entirety to prove the truth of the matter asserted? a. "I feel very sad that I stole your camera." b. "I am very scared because I think my father is going to beat me again." c. "I am very worried about Gloria; I think she has a drinking problem." d. "I am planning on going to school tomorrow to study geology."

d. is correct. Rule 803(3) allows admission of a declarant's state of mind, including a statement about the declarant's future plans. a. is incorrect. Although this appears to be a statement about the declarant's state of mind ("I feel very sad...") the entire statement is not admissible because it includes a statement of memory ("I stole your camera") which is not admissible under Rule 803(3). b. is incorrect. Although this appears to be a statement about the declarant's state of mind ("I am very scared...") the entire statement is not admissible because it includes a statement of belief ("I think my father is going to beat me again") which is not admissible under Rule 803(3). c. is incorrect. Although this appears to be a statement about the declarant's state of mind ("I am very worried...") the entire statement is not admissible because it includes a statement of belief ("I think she has a drinking problem") which is not admissible under Rule 803(3).

Debra was a fourteen-year-old girl who was being treated for severe depression. During her second session with her psychologist, she told him that her stepfather had sexually abused her, and she described the abuse in detail. This information was critical to determining how to treat Debra's depression. The stepfather was arrested and charged with child molestation. Debra refused to testify at the trial. The psychologist was called to the stand to testify about what Debra told her. The stepfather objects to this testimony as hearsay. How should the court rule on this exception? a. The exception for medical treatment or diagnosis does not apply because the psychologist is not a medical doctor. b. The exception for medical treatment or diagnosis does not apply because the statement involves a declaration of fault. c. The exception for medical treatment or diagnosis does not apply because Debra is not unavailable. d. The exception for medical treatment or diagnosis applies and the statement is admissible.

d. is correct. Rule 803(4) generally does not apply to statements of fault, but in some cases (as with statements regarding emotional trauma brought on by family members), statements of fault are pertinent to medical treatment or diagnosis. a. is incorrect. Rule 803(4) applies to statements for mental treatment or diagnosis as well as physical. b. is incorrect. Rule 803(4) generally does not apply to statements of fault, but in some cases (as with statements regarding emotional trauma brought on by family members), statements of fault are pertinent to the specified medical treatment or diagnosis. In most cases, the identification of the perpetrator would not be pertinent to medical treatment or diagnosis, but in cases of mental trauma they may be. c. is incorrect. Rule 803(4)'s applicability is not dependent upon whether the declarant is available.

Greg rented a stand-alone unit from Store-Your-Stuff storage facility in Florida, and he put six boxes containing his rare book collection inside the unit. Two weeks later, a severe rainstorm hit the area, bringing over 3 inches of rain. Because the roofs of the storage units had not been replaced for over twenty years, they could not keep out such a large quantity of rain, and water leaked into Greg's unit, destroying his books. Greg is now suing Store-Your-Stuff, arguing that the facility was negligent in not replacing the roof or taking any other action to prevent leaks. After some investigation, Greg learned that one week after the storm, Store-Your-Stuff installed tarps over all of its units. At trial during his case-in-chief, Greg wants to admit the fact that Store-Your-Stuff put tarps on the roofs of the storage units. Store-Your-Stuff has not yet contested that attaching a tarp to every roof would not be feasible, though Greg suspects that they might do so when they present their defense. How should the court rule on the evidence of the tarps? a. Admit evidence of the tarps to prove that Store-Your-Stuff was negligent. b. Admit the evidence of the tarps, but only to prove feasibility. c. Admit the evidence of the tarps, but only to prove ownership. d. Preclude all evidence of the tarp installation in Greg's case-in-chief.

d. is correct. Store-Your-Stuff's installation of the tarps was a subsequent remedial measure, since it was an action taken by the defendant after the incident which was meant to prevent a similar incident from occurring again. As such, it is barred by Rule 407 if offered to prove negligence. Since this is being offered in Greg's case-in-chief, Greg cannot argue that he is offering the evidence to prove ownership, control, or feasibility, because Store-Your-Stuff has not yet presented its case. a. is incorrect. Store-Your-Stuff's installation of the tarps was a subsequent remedial measure, since it was an action taken by the defendant after the incident which was meant to prevent a similar incident from occurring again. As such, it is barred by Rule 407 if offered to prove negligence. b. is incorrect. Store-Your-Stuff's installation of the tarps was a subsequent remedial measure, since it was an action taken by the defendant after the incident which was meant to prevent a similar incident from occurring again. It is thus barred by Rule 407. Since this is being offered in Greg's case-in-chief, Greg cannot argue that he is offering the evidence to prove ownership, control, or feasibility, because Store-Your-Stuff has not yet presented its case. c. is incorrect. Store-Your-Stuff's installation of the tarps was a subsequent remedial measure, since it was an action taken by the defendant after the incident which was meant to prevent a similar incident from occurring again. It is thus barred by Rule 407. Since this is being offered in Greg's case-in-chief, Greg cannot argue that he is offering the evidence to prove ownership, control, or feasibility, because Store-Your-Stuff has not yet presented its case.

After an argument about overtime pay, Cleon fired his secretary Fiona. Fiona called Cleon a few hours later and said that she had spoken to an attorney and that she intended to sue Cleon and the company for breach of contract. Cleon apologized over the phone and said that he had lost his temper. He then typed out a letter admitting that it was wrong to fire her, and offering to re-hire her and to pay her the overtime she had asked for. He then scanned the letter into his computer and sent her the letter attached to an e-mail. Fiona rejected his offer and sued Cleon, arguing that firing her and refusing to pay her overtime was a breach of their employment contract. She seeks to admit the letter as evidence that it was wrong for Cleon to fire her and that she deserved the overtime payments. The letter is: a. Inadmissible hearsay. b. Inadmissible as a settlement offer. c. Inadmissible because she does not have the original letter, only the scanned version from the e-mail. d. admissible.

d. is correct. The letter is exempted from the hearsay rule because it is a statement made by a party-opponent. Although Rule 408 does bar settlement offers, it only applies to a "disputed claim." At the time that Cleon wrote the letter, he did not dispute anything that Fiona was alleging—in fact, he conceded that she should be re-hired and that she deserved the overtime payments. Thus, Rule 408 does not apply. Although Rule 1002 requires "an original writing," Rule 1003 makes it clear that a "duplicate is admissible to the same extent as the original" unless there is a genuine question about the original's authenticity or it would otherwise be unfair to admit the duplicate. a. is incorrect. The letter is exempted from the hearsay rule because it is a statement made by a party-opponent. b. is incorrect. Although Rule 408 does bar settlement offers, it only applies to a "disputed claim." At the time that Cleon wrote the letter, he did not dispute anything that Fiona was alleging—in fact, he conceded that she should be re-hired and that she deserved the overtime payments. Thus, Rule 408 does not apply. c. is incorrect. Although Rule 1002 requires "an original writing," Rule 1003 makes it clear that a "duplicate is admissible to the same extent as the original" unless there is a genuine question about the original's authenticity or it would otherwise be unfair to admit the duplicate.

Jamie was opening a soda can when it exploded in her hands, causing serious injury. Her husband drove her to the hospital and dropped her off at the emergency room while he parked the car. She walked up to the desk and explained to the employee behind the desk that a soda can had exploded in her hands. Jamie later sued the company that manufactured the soda can, arguing that there was a defect in its design. During her case-in-chief, she called the hospital employee who admitted her to testify about what she (Jamie) said about the injury when she first arrived at the hospital. Jamie seeks to admit her statements to the employee to prove the truth of the matter asserted. Jamie's statements are: a. Inadmissible because she cannot admit her own hearsay statements as evidence. b. Admissible because Jamie was describing her current state of mind when she made the statement. c. Admissible if the hospital employee was a registered nurse or doctor. d. Admissible if her statements were reasonably relevant to her diagnosis or treatment.

d. is correct. These statements are likely admissible under the hearsay exception for medical diagnosis or treatment (Rule 803(4)). In order to meet the requirements of this exception, Jamie must prove that the statement was made for the purposes of medical diagnosis or treatment, is reasonably pertinent to that treatment, and describes her symptoms or the cause of her symptoms. a. is incorrect. There is no rule that prohibits a party from introducing their own hearsay statements, as long as those statements fit into one of the hearsay exceptions. b. is incorrect. Jamie's statements to the hospital employee went beyond her state of mind and described outside events (the soda can exploding) that happened to her. c. Although this statement is likely to be admissible under hearsay exception for medical treatment or diagnosis (Rule 803(4)), the rule contains no requirement that these statements be made to a registered nurse or doctor.

Lisa owns a small antique store, and one night her store was burglarized. After the police arrested a suspect for the crime, she testified in the grand jury about all of the items that were stolen from her store. As part of her testimony, she read from her store's purchasing orders, which stated the amount of money that each item was worth. After the grand jury testimony, the prosecutor gave Lisa a copy of the transcript of her testimony, and she reviewed it and agreed that it was an accurate record of her testimony. A few weeks later, Lisa's store was destroyed in a fire. All of her records were lost, including the purchasing orders that she had read to the grand jury. When the burglary case went to trial, the prosecutor had to prove that the total value of the items stolen was over a certain amount. Unfortunately, Lisa cannot remember the value of each of the items. How can the prosecutor prove these values? a. Call Lisa to the stand and have her testify about the burglary, and then admit the grand jury transcript as a declarant-witness's prior statement under Rule 801(d)(1). b. Call Lisa to the stand and have her lay the foundation for the store's purchasing orders as a business record under Rule 803(6) and then admit the grand jury testimony where she reads from the purchasing orders. c. Call Lisa to the stand and ask her if she remembers the values of the items that were stolen. When she says she does not, ask her if the grand jury transcript would refresh her recollection. When she agrees, show her the grand jury transcript and have her read the transcript to the jury pursuant to Rule 612. d. Call Lisa to the stand and ask her if she remembers the values of the items that were stolen. When she says she does not, ask her if the grand jury transcript is an accurate record of her testimony that she adopted at a time when it was fresh in her memory. When she agrees, have her lay a foundation for the store's purchasing orders as business records under Rule 803(6). Then have her read the grand jury transcript to the jury pursuant to Rule 803(5).

d. is correct. This is a problem with double hearsay—the grand jury testimony is one level of hearsay, and the purchase orders are another level of hearsay. Thus, the prosecutor must first use Rule 803(5) in order to allow the witness to read the grand jury transcript as a recorded recollection. Then, the prosecutor must establish that the hearsay within the grand jury transcript (the value of the items, as asserted by the purchase orders) falls under the business records exception to the hearsay rule. a. is incorrect. Rule 801(d)(1) will only permit a witness' prior statements if they are inconsistent, or in order to rebut an allegation or recent fabrication, or if they are a prior identification. None of these applies in this case. b. is incorrect. Rule 803(6) would allow the business records into evidence if the prosecutor actually had a copy of the business records, but he does not—he only has Lisa's prior statements about what the business records actually said. Thus, this is a problem of double hearsay, and Rule 803(6) only helps out with one level of the double hearsay. c. is incorrect. Rule 612 will allow the prosecutor to show the witness any kind of writing in order to refresh her recollection, but then requires that the writing be taken away from the witness so that she can testify from her refreshed recollection. Thus, she cannot read from the grand jury transcript using Rule 612.

Carl is on trial for murder. He is pleading self-defense. The prosecutor has a character witness who will testify that in his opinion Carl is a vicious individual who routinely uses violence to get his way. Can the prosecutor's character witness testify at Carl's trial? a. No, not under any conditions. b. Yes, the character witness can testify in the prosecutor's case-in-chief. c. Yes, the character witness can testify, but only after the defendant testifies. d. Yes, the character witness can testify, but only after the defendant calls his own character witness.

d. is correct. Under Rule 404(a), the defendant "opens the door" to an attack on his character if he calls a positive character witness. a. is incorrect. In a criminal case, Rule 404(a) sets out certain conditions under which the prosecutor can call a character witness. b. is incorrect. Under Rule 404(a), the prosecutor cannot initiate the use of character witnesses in a criminal case unless the defendant is claiming self-defense in a homicide case and the prosecutor wants to admit evidence of the victim's peaceful character. Here, although the defendant is claiming self-defense in a homicide case, the prosecutor's character evidence is about the defendant's violent nature, not the victim's peaceful nature. c. is incorrect. Under Rule 404(a), the fact that the defendant testifies does not give the prosecutor the right to call a character witness.

A woman was struck by a brick with her name scrawled on it that was thrown through her bedroom window. The victim believes that her ex-boyfriend, who is a gang member, threw the brick because she has become active in anti-gang groups, but she did not actually see him throw it. If the ex-boyfriend is arrested and put on trial for battery, which of the following items of the victim's proposed testimony is LEAST likely to be admitted? Press Enter or Space to submit the answer a. the victim recently move to a new apartment and only her ex-boyfriend and a few family members knew its location b. the victim had testified against a member of her ex-boyfriend's gang last month in a drug case c. on another occasion, the victim had seen her ex-boyfriend throw a rock through the window of a rival street gang member d. immediately after the brick went through her window, the victim heard a voice she recognized as her ex-boyfriend's yell, "If you don't start minding your own business, you'll get a lot worse than this next time!"

"least likely to be admitted" = most likely to be excluded c. is correct. Evidence of the defendant's other crimes or misconduct is admissible only if relevant to some issue other than the defendant's character or propensity to commit the crime charged. Such acts would be admissible to show motive, intent, absence of mistake, identity, or a common plan or scheme. Of these, the only one possibly relevant to these facts is identity. Evidence that the accused committed prior criminal acts that are so distinctive as to operate as a "signature" may be introduced to prove that the accused committed the act in question. Merely throwing an object, such as a brick, through a window could not be considered so distinctive as to operate as a signature. Thus, this evidence would not show identity. The only possible reason for offering the evidence is to show the ex-boyfriend's propensity to commit the crime charged, in which case the testimony will be inadmissible. (A) is wrong because it is circumstantial evidence that the ex-boyfriend threw the brick. It is relevant because it tends to make it more probable that he threw the brick than it would be without the evidence. (B) is wrong because it is relevant and goes to motive. It too makes it more probable that the ex-boyfriend threw the brick than it would be if the victim had not testified against a member of his gang. (D) is wrong because the victim's identification of the ex-boyfriend's voice places him at the scene and is thus relevant. It is more probable that he threw the brick than it would be in the absence of this testimony. The identification of a voice is properly authenticated by the opinion of a person familiar with the alleged speaker's voice. As his ex-girlfriend, the victim would be sufficiently familiar with the ex-boyfriend's voice to make a proper identification.

A plaintiff brings a federal civil rights action against several members of a police department for near-fatal injuries incurred as a result of an alleged beating administered by the officers. The plaintiff plans to call as a witness his cellmate, who will testify that the plaintiff was in fact beaten by the defendants. The plaintiff also wants to introduce the bloodstained shirt that he wore on the night of the beating. The defendants plan to call an expert to testify that the injuries suffered by the plaintiff were inconsistent with injuries likely to be inflicted by the alleged police beating, and a second expert to testify similarly. The defendants also will call as a witness a fellow officer, who will testify that the plaintiff was suspected to have hepatitis, and as a result the defendants would not have beaten him for fear of being infected by any open wounds. Which item of relevant evidence is the court LEAST likely to exclude? a. the cellmate's testimony, on the ground that calling this witness constitutes an unfair surprise b. the bloody shirt, on the ground that it will create a danger of unfair prejudice. c. the second expert's testimony, on the grounds that it will constitute a waste of time and will unnecessarily present cumulative evidence d. the fellow officer's testimony, on the ground that it may confuse the issues or mislead the jury.

"least likely to exclude" = most likely to get in a. is correct. Under Federal Rule 403, a trial judge has broad discretion to exclude relevant evidence if its probative value is substantially outweighed by the danger of unfair prejudice, confusion of the issues, or misleading the jury, or by considerations of undue delay, waste of time, or needless presentation of cumulative evidence. Although some states list unfair surprise as an additional basis for exclusion, the Federal Rules do not, reasoning that surprise can be prevented by discovery and pretrial conference, or mitigated by granting a continuance. From the foregoing principles, if the cellmate is in fact a surprise witness, this will not suffice as a basis to exclude this otherwise relevant evidence under the Federal Rules, which govern this action. At most, the court should grant a continuance. (B) is incorrect because the bloodstained shirt might be deemed to be inflammatory and capable of producing an unfairly prejudicial effect on the jury. As such, it is within the realm of the judge's discretionary power of exclusion. (C) is incorrect because the second expert's testimony will not add anything to the testimony already given by the first expert. Thus, allowing this testimony will simply waste time and repeat evidence already presented. Pursuant to Rule 403, this constitutes a permissible ground of exclusion. (D) is incorrect because, although the fellow officer's testimony tends to render more probably untrue the allegation of a police beating than it would have been without this testimony, whether the plaintiff has hepatitis is not an issue in the case. Thus, the statement referring to the defendants' fears might well cause confusion of the issues or tend to mislead the jury, and is subject to exclusion under the Federal Rules.


Conjuntos de estudio relacionados

Properties Used in Algebra and Geometry/Complex Number System

View Set

Chapter 52 Case Study and Questions

View Set

Individual, Family and Community Health Promotion Ch 6, 7, 8, 9

View Set

Dia de los muertos True or False

View Set

IPC Chapter 1: Science laws and theoires, scientifc method, sigifiant figures, scientific notation and the metric system

View Set